PPQs Flashcards

1
Q

Elderly woman with C. diff colitis and profuse diarrhoea

A

Hypovolaemic hyponataraemia

How well did you know this?
1
Not at all
2
3
4
5
Perfectly
2
Q

Man with cough, SoB and weight loss. Hyponatraemia, low serum osmolality, urine Na and osmolality were inappropriately normal. Diagnosis?

A
  1. SIADH secondary to small cell lung cancer
How well did you know this?
1
Not at all
2
3
4
5
Perfectly
3
Q

Low plasma sodium, low urine sodium

A
  1. Psychogenic polydipsia
How well did you know this?
1
Not at all
2
3
4
5
Perfectly
4
Q

Man who has been in a car accident, raised sodium and plasma osmolality, low urine osmolality

A
  1. Cranial diabetes insipidus
How well did you know this?
1
Not at all
2
3
4
5
Perfectly
5
Q

Raised Na, low K, HTN but raised renin

A
  1. Renal artery stenosis

NOTE: RAISED RENIN MEANS RENAL ARTERY STENOSIS, if this was low would be Conn’s syndrome

How well did you know this?
1
Not at all
2
3
4
5
Perfectly
6
Q

Refractory, elevated levels of what ion are an indication for dialysis?

A

K+

How well did you know this?
1
Not at all
2
3
4
5
Perfectly
7
Q

Woman comes in with bitemporal hemianopia, 2cm mass, and a raised prolactin 1400

A

non functional macroadenomas can compress dopamine stalk and cause raised prolactin, they are most likely to cause visual defects

How well did you know this?
1
Not at all
2
3
4
5
Perfectly
8
Q

Woman comes in with no visual change, 4mm mass, and raised prolactin 1400

A
  1. Prolactinoma
How well did you know this?
1
Not at all
2
3
4
5
Perfectly
9
Q

Raised prolactin, raised TSH, raised T4

A

TSHoma

NOTE: Raised prolactin, raised TSH, raised T4

How well did you know this?
1
Not at all
2
3
4
5
Perfectly
10
Q

calcium high, PTH low

A
  1. Bone metastases
How well did you know this?
1
Not at all
2
3
4
5
Perfectly
11
Q

Calcium low, PTH low

A
  1. Hypoparathyroidism
How well did you know this?
1
Not at all
2
3
4
5
Perfectly
12
Q

Blood levels in osteoporosis

A

Normal

How well did you know this?
1
Not at all
2
3
4
5
Perfectly
13
Q

Order of investigations in acromegaly

A

Serum IGF then OGTT (diagnostic)

How well did you know this?
1
Not at all
2
3
4
5
Perfectly
14
Q

Gold line investigation for cushing’s disease

A

Inferior petrosal sinus sampling, do low dose dexmeth first

How well did you know this?
1
Not at all
2
3
4
5
Perfectly
15
Q

24 year old asymptomatic man with isolated rise in unconjugated bilirubin (all other results (ALT, ALP etc normal)

A
  1. Gilbert’s
How well did you know this?
1
Not at all
2
3
4
5
Perfectly
16
Q
  • 22 year old student with two weeks anorexia, fever and malaise – ALT was most elevated, raised ALP + GGT
A

Viral Hepatitis

NOTE: ALT most elevated indicates viral hepatitis

How well did you know this?
1
Not at all
2
3
4
5
Perfectly
17
Q

A 57 year old man presents following hematemesis and is found to have high ALT + GGT, slightly raised ALP, and low Albumin

A

Cirrhosis

NTOE: albumin low indicates chronic liver disease

How well did you know this?
1
Not at all
2
3
4
5
Perfectly
18
Q

Deficiency of which enzyme leads to hyperuricemia?

A
  1. HGPRT
How well did you know this?
1
Not at all
2
3
4
5
Perfectly
19
Q

Which enzyme regulates the rate limiting step in the haem biosynthesis pathway?

A
  1. ALA synthase
How well did you know this?
1
Not at all
2
3
4
5
Perfectly
20
Q

enzyme that is raised in mumps

A
  1. Amylase-S
How well did you know this?
1
Not at all
2
3
4
5
Perfectly
21
Q

High calcium, normal phosphate, low PTH, high ALP, patient feels tired

A
  1. Metastatic disease
How well did you know this?
1
Not at all
2
3
4
5
Perfectly
22
Q

72 year old with headache: high CA, normal phosphate, normal PTH, high ALP

A
  1. Pagets
How well did you know this?
1
Not at all
2
3
4
5
Perfectly
23
Q

Baby with seizures: low Ca, low PTH

A

Hypoparathyroidism (e.g. du to DiGeorge syndrome)

How well did you know this?
1
Not at all
2
3
4
5
Perfectly
24
Q

Pt presents with low mood after renal transplant following longstanding renal disease: high Ca, high PTH

A

Tertiary hyperparathyroidism

How well did you know this?
1
Not at all
2
3
4
5
Perfectly
25
Q

10 year old with seizures: low Ca, high PO4, high PTH

A

Renal osteodystrophy

How well did you know this?
1
Not at all
2
3
4
5
Perfectly
26
Q

most common cause of acute pancreatitits

A

gallstones

How well did you know this?
1
Not at all
2
3
4
5
Perfectly
27
Q

Most common cause of chronic pancreatitis

A

alcohol

How well did you know this?
1
Not at all
2
3
4
5
Perfectly
28
Q

Which thyroid cancer most commonly metastasises to the lymph nodes?

A

Papillary

How well did you know this?
1
Not at all
2
3
4
5
Perfectly
29
Q

T1DM with hypoglycaemia, what is the management option if no IV access?

A

IM glucagon

How well did you know this?
1
Not at all
2
3
4
5
Perfectly
30
Q

enzyme that makes monosodium urate in gout, needle shaped aspirate with negative birefringence

A
  1. Xanthine oxidase
How well did you know this?
1
Not at all
2
3
4
5
Perfectly
31
Q

cause of dark urine with no blood on microscpy

A
  1. Myoglobin
How well did you know this?
1
Not at all
2
3
4
5
Perfectly
32
Q

Marker for heart failure

A
  1. NT-pro BNP
How well did you know this?
1
Not at all
2
3
4
5
Perfectly
33
Q

liver enzyme raised in heart attack

A

AST

How well did you know this?
1
Not at all
2
3
4
5
Perfectly
34
Q

PCSK9 inhibitor - evolucumab, what does it halve?

A

LDL levels

How well did you know this?
1
Not at all
2
3
4
5
Perfectly
35
Q

Which molecule takes cholesterol and moves it to liver and steroidogenesis organs?

A

HDL

How well did you know this?
1
Not at all
2
3
4
5
Perfectly
36
Q

Vitamin deficiency that causes megaloblastic anaemia and neural tube defect.

A

FOLATE - vitamin B9

How well did you know this?
1
Not at all
2
3
4
5
Perfectly
37
Q

What vitamin is folate?

A

B9

How well did you know this?
1
Not at all
2
3
4
5
Perfectly
38
Q

What herpes virus is EBV?

A

HHV4

How well did you know this?
1
Not at all
2
3
4
5
Perfectly
39
Q
  1. Rank the following in order of highest to lowest reticulocyte count
    a. Anaemia secondary to low dose myelosuppressive chemotherapy
    b. Hereditary spherocytosis
    c. ITP
    d. Occult GI blood loss
    e. Severe aplastic anaemia
A

Hereditary spherocytosis: This is a hemolytic anemia where the bone marrow responds by producing more reticulocytes to replace the destroyed red blood cells. Therefore, it typically has a high reticulocyte count.

Occult GI blood loss: Chronic blood loss from the gastrointestinal tract stimulates the bone marrow to increase reticulocyte production to compensate for the lost red blood cells. Thus, it would have a moderately high reticulocyte count.

ITP (Immune Thrombocytopenic Purpura): This condition primarily affects platelets, but if there is associated significant bleeding and resultant anemia, the bone marrow would respond by increasing reticulocyte production. However, the reticulocyte count would generally be less than in conditions with direct red cell destruction.

Anaemia secondary to low dose myelosuppressive chemotherapy: Chemotherapy suppresses bone marrow activity, leading to reduced production of all blood cells, including reticulocytes. Thus, the reticulocyte count would be low.

Severe aplastic anemia: In this condition, the bone marrow fails to produce sufficient blood cells, including reticulocytes. Therefore, it has the lowest reticulocyte count.

How well did you know this?
1
Not at all
2
3
4
5
Perfectly
40
Q
  1. 62 y/o woman attends GP with tachycardia and fatigue. FBC chows macrocytic anaemia, thrombocytopenia, and neutropenia. The blood film shows neutrophils have reduced granularity and lobation, and there is no polychromasia. Rank the following in likelihood of being the diagnosis:
    a. Aplastic anaemia
    b. Autoimmune thrombocytopenic purpura
    c. B12 deficiency
    d. Chronic lymphocytic leukaemia
    e. Myelodysplastic syndrome
A

Myelodysplastic syndrome (MDS) - MDS often presents with pancytopenia (anemia, thrombocytopenia, neutropenia) and abnormal blood cell morphology, including neutrophils with reduced granularity and lobation. The lack of polychromasia (immature red blood cells) also suggests a problem with bone marrow function typical of MDS.

Aplastic anemia - This condition can also present with pancytopenia, but the blood film in aplastic anemia typically shows very few cells overall, without the specific morphological changes in neutrophils seen in this case.

Chronic lymphocytic leukemia (CLL) - CLL usually presents with lymphocytosis rather than pancytopenia. However, in advanced stages, it can lead to bone marrow failure and pancytopenia. The blood film in CLL typically shows an increased number of mature lymphocytes rather than the described changes in neutrophils.

Autoimmune thrombocytopenic purpura (ITP) - ITP primarily causes isolated thrombocytopenia and would not typically present with anemia and neutropenia or the specific morphological changes in neutrophils.

B12 deficiency - B12 deficiency causes macrocytic anemia and sometimes mild thrombocytopenia and neutropenia. However, it typically presents with hypersegmented neutrophils, not reduced granularity and lobation. Additionally, B12 deficiency usually causes polychromasia due to the ineffective erythropoiesis.

How well did you know this?
1
Not at all
2
3
4
5
Perfectly
41
Q
  1. Rank the cancers below from most to frequent as a cause of death of men in the UK
    a. Breast
    b. Colon
    c. Head and neck
    d. Lung
    e. Prostate
A

Lung
Colon
Prostate
Head and neck
Breast

How well did you know this?
1
Not at all
2
3
4
5
Perfectly
42
Q
  1. Rank the following stages in evolution of oesophageal adenocarcinoma
    a. High grade dysplasia
    b. Adenocarcinoma
    c. Low grade dysplasia
    d. Reactive
    e. Metaplasia
A

Reactive changes (d) - This stage involves the initial response of the esophageal lining to chronic irritation, such as from gastroesophageal reflux disease (GERD). These changes are non-neoplastic and are characterized by inflammation and reactive cellular alterations.

Metaplasia (e) - In this stage, the normal squamous epithelium of the esophagus is replaced by columnar epithelium with goblet cells, known as Barrett’s esophagus. This is a pre-neoplastic condition and a direct response to chronic irritation and inflammation.

Low grade dysplasia (c) - This stage involves early neoplastic changes in the metaplastic epithelium. The cells show abnormal growth and appearance but are not yet highly atypical or invasive.

High grade dysplasia (a) - This stage is characterized by more severe cellular abnormalities and a higher likelihood of progressing to invasive cancer. The cells exhibit significant atypia and architectural distortion.

Adenocarcinoma (b) - This is the final stage where the dysplastic cells have invaded beyond the basement membrane and formed an invasive malignancy.

How well did you know this?
1
Not at all
2
3
4
5
Perfectly
43
Q
  1. 72 y/o Afro-Caribbean woman is admitted with acute SOB. PMHx includes hypertension and T2DM. She takes Metformin, Atorvastatin, and Amlodipine. O/E BP is 148/96. Auscultation reveals a 3rd heart sound and bibasal crackles, but no murmurs.
    Investigations:
    Sodium 142
    Potassium 3.5
    Urea 12.4
    Creatinine 126
    Rank the following diagnoses from most to least likely:
    a. Essential hypertension
    b. Cushing’s syndrome
    c. Conn’s syndrome
    d. Phaeochromocytoma
    e. Addison’s disease
A

Essential hypertension (a) - This is the most common cause of hypertension, especially in an older adult. The patient’s blood pressure is elevated, which can contribute to heart failure (indicated by the third heart sound and bibasal crackles).

Conn’s syndrome (Primary hyperaldosteronism) (c) - This condition involves excessive production of aldosterone, leading to hypertension and hypokalemia. However, the patient’s potassium level is normal, making this less likely but still a consideration due to the hypertension.

Cushing’s syndrome (b) - Cushing’s syndrome can cause hypertension and other metabolic abnormalities. However, it is less common and typically presents with other features such as central obesity, purple striae, and a moon face, which are not mentioned in this case.

Phaeochromocytoma (d) - This is a rare cause of hypertension due to excessive catecholamine secretion from an adrenal tumor. It typically presents with paroxysmal episodes of hypertension, palpitations, sweating, and headache, none of which are mentioned in the history.

Addison’s disease (e) - Addison’s disease typically presents with hypotension, hyponatremia, and hyperkalemia due to adrenal insufficiency. This patient’s blood pressure is elevated, and electrolyte levels are normal, making Addison’s disease the least likely diagnosis.

How well did you know this?
1
Not at all
2
3
4
5
Perfectly
44
Q
  1. Rank the following in order of efficacy at reducing LDL:
    a. Atorvastatin
    b. Bezafibrate
    c. Evolocumab
    d. Prednisolone
    e. Simvastatin
A

Evolocumab (c) - Evolocumab is a PCSK9 inhibitor, which significantly reduces LDL levels, typically by around 60% or more. It is among the most potent agents available for lowering LDL cholesterol.

Atorvastatin (a) - Atorvastatin is a high-potency statin, and it can reduce LDL levels by about 40-60%, depending on the dose.

Simvastatin (e) - Simvastatin is a moderate-potency statin, and it can reduce LDL levels by about 20-40%, depending on the dose.

Bezafibrate (b) - Bezafibrate is a fibrate, which primarily reduces triglycerides but has a modest effect on lowering LDL cholesterol, generally around 5-20%.

Prednisolone (d) - Prednisolone is a corticosteroid and does not lower LDL cholesterol; in fact, it can increase LDL levels.

How well did you know this?
1
Not at all
2
3
4
5
Perfectly
45
Q
  1. 55 y/o woman develops dry mouth and eyes alongside fatigue and arthralgia of the small joints in her hands. Investigations:
    Urate normal
    ESR 64 (raised)
    IgG 22 (raised)
    Rheumatoid factor 120 (raised)
    Anti-CCP antibody 0.9 (negative)
    Speckled anti-nuclear antibody titre 1:640 (raised)
    Rank the following diagnoses by likelihood:
    a. Gout
    b. Osteoarthritis and keratoconjunctivitis sicca
    c. Osteogenesis imperfecta
    d. Primary Sjogren’s syndrome
    e. Rheumatoid arthritis
A

Primary Sjogren’s syndrome (d) - The combination of dry mouth, dry eyes, fatigue, and arthralgia strongly suggests Sjogren’s syndrome. The elevated ESR, IgG, and positive anti-nuclear antibody with a speckled pattern further support this diagnosis. Sjogren’s syndrome often presents with these symptoms and laboratory findings.

Osteoarthritis and keratoconjunctivitis sicca (b) - Osteoarthritis typically presents with joint pain and stiffness, but it does not usually cause systemic inflammation or elevated IgG and ESR. Keratoconjunctivitis sicca (dry eyes) could be present, but it does not explain the other systemic findings.

Rheumatoid arthritis (e) - Although rheumatoid factor is elevated, the negative anti-CCP antibody makes rheumatoid arthritis less likely. RA typically presents with small joint involvement and systemic symptoms, but anti-CCP is a more specific marker for RA.

Gout (a) - Gout usually presents with acute, episodic joint inflammation and is associated with elevated urate levels, which are normal in this case. It does not cause dry mouth and eyes, elevated IgG, or anti-nuclear antibodies.

Osteogenesis imperfecta (c) - Osteogenesis imperfecta is a genetic disorder characterized by brittle bones and frequent fractures. It does not present with dry mouth and eyes, elevated inflammatory markers, or positive autoimmune serology.

How well did you know this?
1
Not at all
2
3
4
5
Perfectly
46
Q
  1. 22 y/o woman presents with mild SLE. Rank the following test results by likelihood of appearing in this case:
    a. Absent IgG
    b. Low complement C3
    c. Positive ANA
    d. Positive C3 nephritic factor
    e. Positive ds-DNA
A

Positive ANA (c) - The antinuclear antibody (ANA) test is the most sensitive for SLE, with a positivity rate of over 95% in patients with the condition. It is almost always positive in cases of SLE.

Positive ds-DNA (e) - Anti-double-stranded DNA (anti-dsDNA) antibodies are highly specific for SLE and are present in about 50-70% of patients with the disease. They are often associated with disease activity and severity, but can also be present in mild cases.

Low complement C3 (b) - Complement levels, particularly C3 and C4, can be low in SLE due to consumption during the immune response. This is more commonly seen in active or severe disease but can also be present in mild cases.

Positive C3 nephritic factor (d) - C3 nephritic factor is associated with certain types of glomerulonephritis and is not commonly seen in SLE. It is less likely to appear in a mild SLE case.

Absent IgG (a) - Absence of IgG is not characteristic of SLE. In fact, patients with SLE often have elevated levels of immunoglobulins due to chronic immune system activation. An absent IgG would suggest a different condition, such as a primary immunodeficiency.

How well did you know this?
1
Not at all
2
3
4
5
Perfectly
47
Q
  1. A 20 y/o woman presents w 2 days of dysuria, increased urinary frequency, and suprapubic pain. Rank the organisms by likelihood of being causative:
    i. Proteus mirabilis
    ii. Acinetobacter baumanii
    iii. Candida albicans
    iv. Escherichia coli
    v. Staphylococcus saprophyticus
A

Escherichia coli (iv) - E. coli is by far the most common cause of uncomplicated UTIs, responsible for 70-90% of cases.

Staphylococcus saprophyticus (v) - This is the second most common cause of UTIs in young sexually active women, accounting for about 5-15% of cases.

Proteus mirabilis (i) - Proteus species are less common but still a notable cause of UTIs, particularly in individuals with urinary tract abnormalities or those who are catheterized.

Candida albicans (iii) - Candida species are typically associated with fungal infections, and Candida albicans is more likely in patients with indwelling catheters, diabetes, or immunosuppression. It is not a common cause of uncomplicated UTIs in otherwise healthy individuals.

Acinetobacter baumanii (ii) - Acinetobacter species are uncommon causes of community-acquired UTIs and are more typically associated with hospital-acquired infections, particularly in patients with prolonged hospitalization or indwelling catheters.

How well did you know this?
1
Not at all
2
3
4
5
Perfectly
48
Q
  1. A neonate develops meningitis at 36 hours old. Rank the organisms by likelihood of being causative:
    i. Escherichia coli
    ii. Group B Strep
    iii. Cryptococcus neoformans
    iv. Pseudomonas aeruginosa
    v. Listeria monocytogenes
A

Group B Streptococcus (ii) - Group B Streptococcus (GBS) is the most common cause of early-onset neonatal meningitis. It is typically acquired during delivery from the mother’s genital tract.

Escherichia coli (i) - E. coli, particularly strains that express the K1 antigen, is another common cause of early-onset neonatal meningitis. It is also acquired from the maternal genital tract during birth.

Listeria monocytogenes (v) - Listeria is a less common cause of neonatal meningitis but is still significant. It can be transmitted vertically from the mother to the neonate during childbirth.

Cryptococcus neoformans (iii) - Cryptococcus is an extremely rare cause of neonatal meningitis and is typically seen in immunocompromised individuals. It is not a typical cause of early-onset neonatal meningitis.

Pseudomonas aeruginosa (iv) - Pseudomonas is a rare cause of neonatal meningitis and is more commonly associated with hospital-acquired infections. It is unlikely in a healthy neonate presenting with early-onset meningitis

How well did you know this?
1
Not at all
2
3
4
5
Perfectly
49
Q
  1. At what temperature can platelets be stored for transfusion for 7 days?
A

20-24 degrees

How well did you know this?
1
Not at all
2
3
4
5
Perfectly
50
Q

Name of LMWH

A

Enoxaparin

How well did you know this?
1
Not at all
2
3
4
5
Perfectly
51
Q

What does this image show?

A

Reed sternberg cells - owel eyes

How well did you know this?
1
Not at all
2
3
4
5
Perfectly
52
Q
  1. 45 y/o man attends GP with fatigue and easy bruising. He has recently returned from India where he had a self-resolving episode of jaundice and malaise. Blood tests show a pancytopenia with low reticulocytes. What is the most likely diagnosis?
A

Aplastic anaemia due to parvovirus B19

How well did you know this?
1
Not at all
2
3
4
5
Perfectly
53
Q
  1. What naturally occurring antibody will be in the serum of an A negative person?
A

Anti-B IgM

How well did you know this?
1
Not at all
2
3
4
5
Perfectly
54
Q
  1. An O negative pregnant woman undergoes amniocentesis. What intervention does this indicate?
A

Anti-D immunoglobulin

How well did you know this?
1
Not at all
2
3
4
5
Perfectly
55
Q
  1. Venetoclax is a BCL2 inhibitor used for B-cell CLL. Which cellular process does this act on?
A

Venetoclax acts on the cellular process of apoptosis

How well did you know this?
1
Not at all
2
3
4
5
Perfectly
56
Q
  1. A 55 year old man attends his GP with severe fatigue and back pain. Investigations:
    Creatinine 635 (high)
    IgG 3.5 (low)
    IgM 0.3 (low)
    Serum protein electrophoresis – no paraprotein
    Kappa light chains 0.9 (low)
    Lambda light chain 1970 (very high)
    K/L ratio 0.0005 (low)
    What is the cause of the renal failure?
A

Monoclonal gammopathy of renal significance

NOTE: Elevated Creatinine: The elevated creatinine level indicates renal dysfunction.

Low IgG and IgM: While not specific, low levels of immunoglobulins (IgG and IgM) can be seen in disorders associated with dysregulation of the immune system, such as monoclonal gammopathies.

Elevated Lambda Light Chains: The markedly elevated lambda light chain level and very low kappa light chain level, along with a significantly decreased kappa/lambda (K/L) ratio, indicate monoclonal gammopathy.

No Paraprotein Detected: The absence of a detectable paraprotein on serum protein electrophoresis does not rule out monoclonal gammopathy, as some patients with LCPT may not have detectable paraproteins.

How well did you know this?
1
Not at all
2
3
4
5
Perfectly
57
Q
  1. Why is sickle cell anaemia not symptomatic for the first 3 months of life?
A

sickle cell isn’t symptomatic till > 3months as fetal Hb deosn’t contain beta Hb chains

How well did you know this?
1
Not at all
2
3
4
5
Perfectly
58
Q
  1. Most common cause of constrictive pericarditis in developing world?
A

Tuberculosis

How well did you know this?
1
Not at all
2
3
4
5
Perfectly
59
Q
  1. What is the most common cause of myocarditis
A

Viral infection (coxsackie B)

How well did you know this?
1
Not at all
2
3
4
5
Perfectly
60
Q
  1. What is the most common cause of portal vein thrombosis
A

Cirrhosis of the liver

How well did you know this?
1
Not at all
2
3
4
5
Perfectly
61
Q
  1. What is the most common primary breast cancer?
A

invasive ductal carcinoma

How well did you know this?
1
Not at all
2
3
4
5
Perfectly
62
Q

Most common type of oesophagael cancer in the UK

A

Adenocarcinoma - Barrets
SCC - smoking - superfical (most common worldwide)

How well did you know this?
1
Not at all
2
3
4
5
Perfectly
63
Q
  1. What is the most likely cause of a mobile mass in a young woman’s breast?
A

Fibroadenoma (aka breast mouse)

How well did you know this?
1
Not at all
2
3
4
5
Perfectly
64
Q
  1. Which underlying pathology is commonly associated with nephrotic syndrome in adults
A

Membranous glomerulonephritis

How well did you know this?
1
Not at all
2
3
4
5
Perfectly
65
Q
  1. What is the most common primary tumour of the heart?
A

Myxoma

How well did you know this?
1
Not at all
2
3
4
5
Perfectly
66
Q
  1. What is the equivalent of a dysgerminoma in a testis?
A

Seminoma

How well did you know this?
1
Not at all
2
3
4
5
Perfectly
67
Q
  1. Intestinal metaplasia in Barrett’s oesophagus is most commonly due to the presence of which cell?
A

Simple columnar epithelial cells

How well did you know this?
1
Not at all
2
3
4
5
Perfectly
68
Q
  1. What type of emphysema is associated with smoking and chronic bronchitis?
A

Centrilobular (centriacinar) emphysema

How well did you know this?
1
Not at all
2
3
4
5
Perfectly
69
Q
  1. Which vascular tumour is associated with HHV8?
A

Kaposi sarcoma

How well did you know this?
1
Not at all
2
3
4
5
Perfectly
70
Q
  1. Which virus causes encephalitis that particularly affects the temporal lobes?
A

HSV 1

How well did you know this?
1
Not at all
2
3
4
5
Perfectly
71
Q

What is the most common cause of hypocalcaemia in the community? What is the most common cause of hypercalcaemia in the community

A

Vit D deficiency, Hypereparathyroidism

How well did you know this?
1
Not at all
2
3
4
5
Perfectly
72
Q
  1. Which blood test may confirm a diagnosis of acute pancreatitis?
A

Diagnosis ofa cute pancreatitis MAY be confirmed by amylase, but definitely confirmed with lipase (more sensitive)

How well did you know this?
1
Not at all
2
3
4
5
Perfectly
73
Q
  1. Deficiency of which plasma protein occurs in patients with movement disorder and liver disease?
A

Careulopasmin

How well did you know this?
1
Not at all
2
3
4
5
Perfectly
74
Q
  1. Name a hormone that increases urinary phosphate excretion
A

PTH - phosphate trashing hormone

How well did you know this?
1
Not at all
2
3
4
5
Perfectly
75
Q
  1. Which liver enzyme is associated with obstructive jaundice?
A

ALP

How well did you know this?
1
Not at all
2
3
4
5
Perfectly
76
Q
  1. Which condition occurs in MEN 1 and 2a?
A

Parathyroid adenoma

How well did you know this?
1
Not at all
2
3
4
5
Perfectly
77
Q
  1. A patient presents in Addisonian crisis with a systolic BP of 90, which fluid should be given?
A

0.9% saline

How well did you know this?
1
Not at all
2
3
4
5
Perfectly
78
Q
  1. A hypertensive patient has hypokalaemia and raised aldosterone with suppressed Renin, what is the likely diagnosis?
A

Conn’s syndrome

NOTE: Raised renin would be renal artery stenosis

How well did you know this?
1
Not at all
2
3
4
5
Perfectly
79
Q
  1. Which vitamin deficiency causes pellagra?
A

B3 - Niacin

How well did you know this?
1
Not at all
2
3
4
5
Perfectly
80
Q
  1. Which hormone released from fat cells has hypothalamic receptors?
A

The hormone released from fat cells (adipocytes) that acts on hypothalamic receptors is leptin.

NOTE: Leptin is a hormone primarily synthesized and secreted by adipocytes in response to energy stores in the body. Its levels increase with the accumulation of fat and decrease with fat loss. Leptin acts on receptors in the hypothalamus, specifically in the arcuate nucleus, to regulate appetite, energy expenditure, and body weight.

In the hypothalamus, leptin inhibits hunger (appetite suppression) and stimulates energy expenditure, thereby playing a crucial role in the regulation of body weight and energy balance. Leptin signaling in the hypothalamus helps to maintain homeostasis by modulating food intake and energy expenditure in response to changes in adipose tissue mass and energy stores.

How well did you know this?
1
Not at all
2
3
4
5
Perfectly
81
Q
  1. A 35 y/o woman presents with neck pain worse on swallowing, she had an URTI 2 weeks prior. Investigations show low TSH, and high T3 and T4. Technetium scanning shows low thyroid uptake. What is the likely diagnosis?
A

Viral thyroiditis - low thyroid uptake

How well did you know this?
1
Not at all
2
3
4
5
Perfectly
82
Q
  1. Which enzyme level or activity should be measured before giving azathioprine?
A

TPMT

How well did you know this?
1
Not at all
2
3
4
5
Perfectly
83
Q
  1. A young man with periodic fevers has an MEFV mutation. What is the diagnosis?
A

FMF

How well did you know this?
1
Not at all
2
3
4
5
Perfectly
84
Q
  1. What condition does JC virus reactivation cause in an immunosuppressed person?
A

PMFL

How well did you know this?
1
Not at all
2
3
4
5
Perfectly
85
Q
  1. CAR-T cells against CD19 treat haematological malignancy of what cell type?
A

B cell

How well did you know this?
1
Not at all
2
3
4
5
Perfectly
86
Q
  1. Which condition features involvement of kidneys and upper and lower airways, and is associated with cANCA with cytoplasmic staining pattern and specificity for proteinase 3?
A

Granulomatosis with polyangiitis

How well did you know this?
1
Not at all
2
3
4
5
Perfectly
87
Q
  1. A 3 y/o child with recurrent bacterial and fungal infections has a negative dihydrorhodamine test (does not oxidise). What is the diagnosis?
A

Chronic granulomatous disease

How well did you know this?
1
Not at all
2
3
4
5
Perfectly
88
Q
  1. CD40 mutation is associated with which immunodeficiency?
A

Hyper IgM syndrome

How well did you know this?
1
Not at all
2
3
4
5
Perfectly
89
Q
  1. What is the name of a fully differentiated B-cell that makes antibodies?
A

Plasma cell

How well did you know this?
1
Not at all
2
3
4
5
Perfectly
90
Q
  1. What type of lymphocyte is deficient in an individual with bare lymphocyte syndrome and an HLA class II mutation?
A

CD4

How well did you know this?
1
Not at all
2
3
4
5
Perfectly
91
Q
  1. A 5 m/o baby has recurrent infections and an IL2RG (interleukin 2 common gamma chain mutation), what is the diagnosis?
A

X linked SCID

How well did you know this?
1
Not at all
2
3
4
5
Perfectly
92
Q
  1. 29 y/o man returns from charity work in Yemen with profuse watery diarrhoea that “looks like water after you cook rice in it”. What is the causative organism?
A

Vibrio Cholera

How well did you know this?
1
Not at all
2
3
4
5
Perfectly
93
Q
  1. 43 y/o receiving chemotherapy for leukaemia has ongoing fever and raised inflammatory markers after broad spectrum antibiotic therapy with Meropenem and Amikacin. CT chest shows nodules with surrounding hypoattenuation (halo signs). What is the most likely causative organism?
A

Aspergillus fumigatus

How well did you know this?
1
Not at all
2
3
4
5
Perfectly
94
Q
  1. 18 y/o man with 1 day history of headache, neck stiffness, and photophobia, but no confusion or other signs of encephalitis. HSV1 detected in CSF PCR. What antiviral treatment would you recommend?
A

Aciclovir

How well did you know this?
1
Not at all
2
3
4
5
Perfectly
95
Q
  1. 42 y/o man returns from India with high fever and abdo pain but no diarrhoea. Blood cultures grow Gram negative bacilli and Malaria rapid test is negative. What is the most appropriate initial antibiotic therapy?
A

Ceftriaxone

How well did you know this?
1
Not at all
2
3
4
5
Perfectly
96
Q
  1. Which Herpes virus causes post-transplant lymphoproliferative disorder?
A

EBV (AKA HHV4)

How well did you know this?
1
Not at all
2
3
4
5
Perfectly
97
Q
  1. 37 y/o woman with hot swollen, painful left knee has Gram positive cocci in clusters on joint aspirate. NKDA and MRSA screen is negative: which narrow spectrum antibiotic is most appropriate here?
A

Fluclox

How well did you know this?
1
Not at all
2
3
4
5
Perfectly
98
Q
  1. This vaccine-preventable disease presents with headache, fever, and parotid swelling. Can cause epididymo-orchitis in adults and sometimes even meningitis. What is the causative organism?
A

Mumps orthorubulavirus

How well did you know this?
1
Not at all
2
3
4
5
Perfectly
99
Q
  1. 73 y/o woman with headache, confusion, photophobia, and fever. Gram positive rods are found in her CSF, what is the likely causative organism?
A

Listeria Monocytogenes

How well did you know this?
1
Not at all
2
3
4
5
Perfectly
100
Q
  1. Name an antibiotic class with concentration-dependent killing and prolonged persistent effects where the goal is to maximise the concentration above the Minimum Inhibitory Concentration (Peak>MIC)
A

Aminoglycosides

How well did you know this?
1
Not at all
2
3
4
5
Perfectly
101
Q
  1. 78 y/o woman has loose green mucoid stool after a week of I.V. Ceftriaxone. What is the causative organism?
A
  1. Clostridium Difficile
How well did you know this?
1
Not at all
2
3
4
5
Perfectly
102
Q
  1. 64 y/o man with poorly controlled T2DM develops periorbital oedema, headache, sinus pain, purulent nasal discharge, and orbital cellulitis. The symptoms started yesterday and he is taken to theatre as an emergency. What anti-fungal should be started immediately?
A
  1. Amphotericin B
How well did you know this?
1
Not at all
2
3
4
5
Perfectly
103
Q
  1. Which stage of pregnancy has the highest risk of VTE?
    a. 1st trimester
    b. Post-amniocentesis
    c. 2nd trimester
    d. 3rd trimester
    e. Post-partum
A

Post partum

How well did you know this?
1
Not at all
2
3
4
5
Perfectly
104
Q
  1. Which of the following is not usually found in multiple myeloma?
    a. Anaemia
    b. Humoral immune dysfunction
    c. Osteolytic bone lesions
    d. Renal impairment
    e. Splenomegaly
A

SPLENOMEGALY

How well did you know this?
1
Not at all
2
3
4
5
Perfectly
105
Q
  1. What cell type mediates acute graft vs. host disease in allogeneic stem cell transplant?
    a. Donor B cells
    b. Donor T cells
    c. Recipient B cells
    d. Recipient NK cells
    e. Recipient T cells
A

Donor T cells

How well did you know this?
1
Not at all
2
3
4
5
Perfectly
106
Q
  1. What is the first-line treatment for chronic phase CML?
    a. Ibrutinib (Bruton tyrosine kinase inhibitor)
    b. Imatinib (ABL 1 tyrosine kinase inhibitor)
    c. Rituximab (Anti-CD20 antibody)
    d. Thalidomide (Anti-angiogenic)
    e. Toilizumab (Anti-IL6 antibody)
A

Imatinib

How well did you know this?
1
Not at all
2
3
4
5
Perfectly
107
Q
  1. 55 y/o starts Rivaroxoban, what monitoring is required?
    a. Anti XA assay
    b. aPTT
    c. INR
    d. No monitoring
    e. Platelet count
A

DOACS need no monitoring

How well did you know this?
1
Not at all
2
3
4
5
Perfectly
108
Q
  1. A PET scan of a Hodgkin lymphoma patient shows involvement of nodes in the supraclavicular fossa, mediastinum, inguinal region, and of the spleen. What stage disease is this?
    a. Stage 0
    b. Stage 1
    c. Stage 2
    d. Stage 3
    e. Stage 4
A

Stage 3 - spleen doesn’t count for stage 4

How well did you know this?
1
Not at all
2
3
4
5
Perfectly
109
Q
  1. Which of the following describes the pathophysiology of beta thalassaemia major?
    a. Decreased alpha globin chain production
    b. Decreased beta globin chain production
    c. Decreased gamma globin chain production
    d. Increased alpha globin chain production
    e. Increased beta globin chain production
A

beta thalassaemia major has decreased beta globin chain production

How well did you know this?
1
Not at all
2
3
4
5
Perfectly
110
Q
  1. 73 y/o woman presents to GP with fever and productive cough. Auscultation reveals a pleural rub. Her WCC is elevated at 15 and there is neutrophilia. Her haemoglobin is slightly low at 110, CRP is elevated at 98, and platelets are 210. The blood film shows left-shift and toxic granulation. What is the most likely diagnosis?
    a. Acute myeloid leukaemia
    b. Aplastic anaemia
    c. Chronic myeloid leukaemia
    d. Infectious mononucleosis
    e. Reactive neutrophilia
A

Reactive neutrophillia, common finding in viral infections such as infectious mononucleosis

How well did you know this?
1
Not at all
2
3
4
5
Perfectly
111
Q
  1. A bleeding patient has a prolonged aPTT and PT, but normal platelets and fibrinogen. Which blood component should they be given?
    a. Albumin
    b. Anti-D
    c. Cryoprecipitate
    d. Fresh frozen plasma
    e. Platelets
A

Fresh frozen plasma contains various clotting factors, including factors II (prothrombin), V, VII, VIII, IX, X, XI, and XII, as well as proteins C and S. It is used to correct coagulation factor deficiencies in patients with bleeding disorders or those requiring urgent reversal of anticoagulant effects.

How well did you know this?
1
Not at all
2
3
4
5
Perfectly
112
Q
  1. A 75 y/o woman develops neutropenic sepsis secondary to myelodysplasia. Her FBC is significantly worse than 6 weeks prior, with marked pancytopenia. Blood film shows numerous large cells of primitive appearance. What is the most likely diagnosis?
    a. Development of aplastic anaemia
    b. Development of iron deficiency
    c. Progression to acute myeloid leukaemia
    d. Progression to chronic myeloid leukaemia
    e. Progression to myelofibrosis
A

Progression to AML

Myelodysplastic syndromes (MDS) are a group of clonal hematopoietic stem cell disorders characterized by ineffective hematopoiesis leading to peripheral cytopenias. MDS can progress to acute myeloid leukemia (AML) in some cases, particularly when there is increased blast percentage (>20%) in the bone marrow or peripheral blood. The development of neutropenic sepsis and worsening pancytopenia may indicate disease progression to AML.

How well did you know this?
1
Not at all
2
3
4
5
Perfectly
113
Q
  1. Which of the following decreases during pregnancy?
    a. Factor 8
    b. Fibrinogen
    c. Plasminogen activator inhibitor 1
    d. Protein S
    e. Von Willebrand factor
A

Protein S decreases in pregnancy

How well did you know this?
1
Not at all
2
3
4
5
Perfectly
114
Q
  1. 55 y/o male smoker on furosemide has the following test results:
    Hb 178 (high)
    Hct 51% (high
    Red cell mass 25 (low end of normal)
    Plasma volume 26 (low)
    What is the most likely diagnosis?
    a. High affinity haemoglobinipathy
    b. Polycythaemia vera
    c. Primary polycythaemia
    d. Relative polycythaemia
    e. Secondary polycythaemia
A

D – low red cell mass and low plasma volume – likely relative polycythaemia due to diuretic use

How well did you know this?
1
Not at all
2
3
4
5
Perfectly
115
Q
  1. A 25 y/o woman with acute leukaemia needs an allogeneic stem cell transplant. She is of mixed Afro-Caribbean and European heritage with one sibling – what is the chance of that sibling being HLA identical?
    a. 1:1
    b. 1:1000
    c. 1:2
    d. 1:4
    e. 1:100
A

siblings have 1 in 4 chance of being HLA identical

How well did you know this?
1
Not at all
2
3
4
5
Perfectly
116
Q
  1. Which virus is associated with nasopharyngeal carcinoma?
    a. Epstein-Barr virus
    b. Hepatitis B virus
    c. Hepatitis C virus
    d. HHV8
    e. HPV16
A

EBV

How well did you know this?
1
Not at all
2
3
4
5
Perfectly
117
Q
  1. What is the most common type of skin cancer?
    a. Basal cell carcinoma
    b. Keratoacanthoma
    c. Melanoma
    d. Metastatic cancer
    e. Squamous cell carcinoma
A

BCC

How well did you know this?
1
Not at all
2
3
4
5
Perfectly
118
Q
  1. An extradural/ epidural haemorrhage is caused by damage to which blood vessel?
    a. Berry aneurysm
    b. Bridging veins
    c. Internal carotid
    d. Middle cerebral artery
    e. Middle meningeal artery
A

Middle meningeal artery

How well did you know this?
1
Not at all
2
3
4
5
Perfectly
119
Q
  1. Which type of necrosis is associated with a myocardial infarction?
    a. Abscess formation
    b. Caseous necrosis
    c. Coagulative necrosis
    d. Fat necrosis
    e. Liquefactive necrosis
A

coagulative necrosid

How well did you know this?
1
Not at all
2
3
4
5
Perfectly
120
Q
  1. What is the most common glial cell of the CNS?
    a. Astrocytes
    b. Endothelial cells
    c. Ependymal cells
    d. Microglia
    e. Oligodendrocytes
A

Astrocytes

How well did you know this?
1
Not at all
2
3
4
5
Perfectly
121
Q
  1. What is the most common cause of mitral valve stenosis?
    a. Congenital
    b. Infective endocarditis
    c. Pulmonary hypertension
    d. Rheumatic heart disease
    e. Systemic lupus
A

Rheumatic heart disease

How well did you know this?
1
Not at all
2
3
4
5
Perfectly
122
Q
  1. What is the mechanism behind pulmonary oedema due to liver disease?
    a. Decreased osmotic pressure
    b. Direct injury to the alveolar wall
    c. Increased hydrostatic pressure
    d. Indirect injury to the alveolar wall
    e. Lymphatic obstructio
A

Pulmonary edema due to liver disease, known as hepatopulmonary syndrome (HPS), primarily occurs as a consequence of the altered pulmonary vascular dynamics resulting from liver dysfunction. The correct mechanism is:

c. Increased hydrostatic pressure.

In liver disease, particularly in cirrhosis, there is increased resistance to blood flow through the liver, leading to portal hypertension. This elevated pressure in the portal venous system causes congestion and dilation of the pulmonary vasculature, increasing hydrostatic pressure within the pulmonary capillaries. As a result, fluid transudates from the capillaries into the interstitial space of the lungs, leading to pulmonary edema.

How well did you know this?
1
Not at all
2
3
4
5
Perfectly
123
Q
  1. Which of the following occurs near the surface of the brain and is frequently asymptomatic?
    a. Glioblastoma multiforme
    b. Haemangioma
    c. Meningioma
    d. Oligodendroglioma
    e. Schwannoma
A

Meningioma

How well did you know this?
1
Not at all
2
3
4
5
Perfectly
124
Q
  1. What is the most common cause of acute respiratory distress syndrome?
    a. Aspiration
    b. Drug reaction
    c. Pancreatitis
    d. Sepsis
    e. Trauma
A

Sepsis

How well did you know this?
1
Not at all
2
3
4
5
Perfectly
125
Q
  1. Which valve is classically involved in infectious endocarditis in I.V. drug users?
    a. Aortic
    b. Mitral
    c. Prosthetic
    d. Pulmonary
    e. Tricuspid
A

Tricuspid

How well did you know this?
1
Not at all
2
3
4
5
Perfectly
126
Q
  1. Which of the following is a characteristic of hypercalcaemia?
    a. Diarrhoea
    b. Easy bruising
    c. Hypotension
    d. Polyuria and polydipsia
    e. Tetany
A

Polyuria and polydipsia

How well did you know this?
1
Not at all
2
3
4
5
Perfectly
127
Q
  1. 25 y/o man brought to A&E with abdominal pain followed by collapse. His blood pressure shows he is in shock and his blood gas results are:
    Na+ 120 (low)
    K+ 6.2 (high)
    HCO3 10.1 (low)
    Urea 9.4 (high)
    Creatinine 146 (high)
    Glucose 2.5 (low)
    pH 7.2 (low)
    pCO2 3.0 (low)
    What is the most likely cause of the abnormal potassium?
    a. Renal loss of sodium
    b. Vomiting
    c. Renal failure
    d. Perforated appendix
    e. Dehydration
A

Renal failure

How well did you know this?
1
Not at all
2
3
4
5
Perfectly
128
Q
  1. Which of the following is found in haemolytic jaundice?
    a. Raised AST
    b. Raised CK
    c. Normal bilirubin
    d. Pale stools
    e. High urobilinogen in urine
A

High urobilinogen in urine

In hemolytic jaundice, there is an increased breakdown of red blood cells (hemolysis), leading to an elevated production of bilirubin. Bilirubin is a breakdown product of heme from hemoglobin and is normally conjugated in the liver and excreted in bile. However, in cases of increased hemolysis, the liver may become overwhelmed, leading to an accumulation of unconjugated bilirubin in the bloodstream.

As a result of increased bilirubin production, there is an elevation in the levels of bilirubin in the blood (hyperbilirubinemia), leading to jaundice (yellowing of the skin and eyes). Additionally, excess unconjugated bilirubin may be excreted in the urine as urobilinogen, resulting in high levels of urobilinogen in the urine (urobilinogenuria).

How well did you know this?
1
Not at all
2
3
4
5
Perfectly
129
Q
  1. Increased insulin sensitivity causes low plasma glucose, and features in which of the following?
    a. Acromegaly
    b. ACTH deficiency
    c. Cushing’s disease
    d. Phaeochromocytoma
    e. PCOS
A

ACTH deficiency

In ACTH deficiency, also known as secondary adrenal insufficiency, there is a deficiency of adrenocorticotropic hormone (ACTH) from the pituitary gland, leading to decreased cortisol production by the adrenal glands. Cortisol is a counter-regulatory hormone to insulin, meaning it opposes the action of insulin and helps maintain blood glucose levels. In the absence of cortisol due to ACTH deficiency, there is decreased gluconeogenesis (glucose production) and increased insulin sensitivity, which can result in lower plasma glucose levels, leading to hypoglycemia.

How well did you know this?
1
Not at all
2
3
4
5
Perfectly
130
Q
  1. Which of the following stimulates receptors in the adrenals and leads to aldosterone release?
    a. ACTH
    b. Angiotensin 2
    c. Calcium
    d. Renin
    e. Sodium
A

Angiotensin 2

How well did you know this?
1
Not at all
2
3
4
5
Perfectly
131
Q
  1. Which of the following can cause hypoglycaemia?
    a. Atorvastatin
    b. Bendrofluazide
    c. Glucagon
    d. Prednisolone
    e. Quinine
A

Quinine

132
Q
  1. Which of the following findings is consistent with moderate alcohol intake?
    a. Reduced albumin
    b. Elevated HDL
    c. Nearly normal AST
    d. Normal GGT
    e. Normal triglycerides
A

Elevated HDL - fatty liver disease

133
Q
  1. 40 y/o woman has post-partum haemorrhage and is then unable to breastfeed. Investigations:
    Cortisol <50 (very low)
    ACTH <10 (very low)
    Prolactin <50 (very low)
    TSH 0.9 (normal)
    Free T4 12 (normal)
    What is the most likely diagnosis?
    a. Acromegaly
    b. Hypopituitarism
    c. Non-functioning macrodenoma
    d. Prolactinoma
    e. TSHoma
A

Hypopituitarism - Sheehan’s

134
Q
  1. Which of the following would be consistent with obstructive jaundice?
    a. Normal ALP
    b. Normal AST
    c. Normal GGT
    d. Dark stools
    e. Increased bilirubin in the urine
A

d. Dark stools

Obstructive jaundice occurs when there is a blockage in the bile ducts, impairing the flow of bile from the liver into the intestines. As a result, bilirubin, a component of bile, cannot be excreted efficiently, leading to its accumulation in the bloodstream and subsequent jaundice.

135
Q
  1. In histology of antibody-mediated rejection of allogeneic kidney transplant, where would the inflammatory infiltrate be seen?
    a. Adipose tissue
    b. Capillaries
    c. Capsule
    d. Interstitium
    e. Tubules
A

d. Interstitium

In AMR, there is an immune response against the transplanted kidney mediated by donor-specific antibodies (DSAs). These antibodies can bind to antigens expressed on the endothelial cells of the renal vasculature, leading to complement activation, inflammation, and tissue injury.

The primary site of injury in AMR is the renal interstitium, which is the space between the renal tubules and contains blood vessels, connective tissue, and immune cells. Inflammatory infiltrates, consisting of lymphocytes, macrophages, and other immune cells, can be observed in the interstitium of the kidney on histological examination.

136
Q
  1. Which of the following is vaccinated against with a conjugate vaccine with a polysaccharide and protein carrier?
    a. HIV
    b. M. tuberculosis
    c. Rabies
    d. S. pneumoniae
    e. V. cholerae
A

Strep pneuumoniae

137
Q
  1. Which of the following cells exists in an immature form in the periphery where they recognise pathogens, then mature and become adapted for presenting antigen to T-cells In lymph nodes?
    a. B lymphocytes
    b. Dendritic cells
    c. Eosinophils
    d. Erythrocytes
    e. Plasma cells
A

Dendritic cells

138
Q
  1. Which of the following enhances T-cell immunity and may be used to treat malignancy?
    a. Infliximab (anti TNFa)
    b. Pembrolizumab (anti PD1)
    c. Rituximab (anti CD20)
    d. Tocilizumab (anti IL6)
    e. Ustekinumab (anti IL12/23
A

b. Pembrolizumab (anti PD1)

Pembrolizumab is a monoclonal antibody that targets the programmed cell death protein 1 (PD-1) receptor on T cells. By blocking the interaction between PD-1 and its ligands (PD-L1 and PD-L2), pembrolizumab enhances T-cell immunity and promotes anti-tumor responses. It is commonly used in the treatment of various malignancies, including melanoma, non-small cell lung cancer, and others.

139
Q
  1. Which of the following disease may respond to plasmapheresis?
    a. Ankylosing spondylitis
    b. Goodpasture syndrome
    c. IgA vasculitis
    d. Primary Sjogren’s syndrome
    e. Takayasu’s arteritis
A

asmapheresis, also known as therapeutic plasma exchange, is a procedure used to remove harmful substances from the blood. It involves separating plasma from blood cells, removing the plasma, and then reinfusing the blood cells back into the patient.

Goodpasture syndrome is an autoimmune disorder characterized by the presence of antibodies that target the basement membrane in the lungs and kidneys, leading to pulmonary hemorrhage and glomerulonephritis. Plasmapheresis can be used as a treatment for Goodpasture syndrome to remove circulating autoantibodies and other inflammatory mediators from the bloodstream, thereby reducing organ damage.

140
Q
  1. Failure to regulate cryopyrin-driven neutrophil activation is characteristic of which disease?
    a. Autoimmune lymphoproliferative syndrome
    b. Behcet’s syndrome
    c. Familial Mediterranean fever
    d. Microscopic polyangiitis
    e. Ulcerative colitis
A

FMF

141
Q
  1. Which of the following is a standard immunosuppressive regime post-allograft?
    a. Azathioprine, mycophenolate mofetil, prednisolone
    b. Cyclophosphamide, methotrexate, rituximab
    c. Cyclosporin, rapamycin, tacrolimus
    d. Dapsone, methotrexate, prednisolone
    e. Mycophenolate mofetil, prednisolone, tacrolimus
A

E - Mycophenolate mofetil (or mycophenolic acid) is a commonly used immunosuppressant that inhibits the proliferation of T and B lymphocytes by blocking de novo purine synthesis. Prednisolone is a corticosteroid that suppresses inflammation and immune responses. Tacrolimus (or cyclosporine) is a calcineurin inhibitor that inhibits T-cell activation and cytokine production.

142
Q
  1. Which of the following drugs can be used to manage Rheumatoid arthritis?
    a. Adalimumab (anti-TNFa)
    b. Basiliximab (Anti-CD25)
    c. Denosumab (anti-RANKL)
    d. Pembrolizumab (anti-PD1)
    e. Secukinumab (anti-IL17)
A

A

143
Q
  1. Which of the following tests is useful in diagnosing invasive Candida albicans infection?
    a. Beta-D-Glucan
    b. Galactomannan
    c. RPR
    d. TPPA
    e. Widal test
A

Beta-D glucan

144
Q

What test may be useful in aspergillus diagnosis?

A

Galactomannan

145
Q
  1. Which of the following is the most appropriate antibiotic for Legionella pnemophilia pneumonia?
    a. Amoxicillin
    b. Azithromycin
    c. Ceftriaxone
    d. Co-trimoxazole
    e. Meropenem
A

Macrolide –> azithromycin

146
Q
  1. Which virus is associated with severe hepatic disease and potentially death if it infects a pregnant patient?
    a. Hepatitis A
    b. Hepatitis B
    c. Hepatitis C
    d. Hepatitis D
    e. Hepatitis E
A

Hepatitis E

Hepatitis E virus (HEV) infection can lead to severe hepatic disease, particularly in pregnant patients. Pregnant women infected with hepatitis E have an increased risk of developing fulminant hepatic failure, which can be life-threatening for both the mother and the fetus. Hepatitis E is particularly problematic in regions with poor sanitation and hygiene, where it is often transmitted through contaminated water.

147
Q
  1. What is the most common form of prion disease?
    a. Fatal familial insomnia
    b. Iatrogenic Creutzfeldt-Jakob disease
    c. Kuru
    d. Sporadic Creutzfeldt-Jakob disease
    e. Variant Creutzfeldt-Jakob disease
A

Sporadic

148
Q
  1. Streptobacillus moniliformis causes which infection?
    a. Bacillary angiomatosis
    b. Lyme disease
    c. Q fever
    d. Rat bite fever
    e. Syphilis
A

Q fever

149
Q
  1. A 45 y/o woman with malaria has a GCS of 11, temp 39.1, HR 108, BP 90/60. Her parasitaemia is 10%, what is the most appropriate anti-malarial therapy?
    a. Intravenous Artesunate
    b. Intravenous Quinine
    c. Oral artemether and proguanil
    d. Oral atorvaquone and proguanil
    e. Oral mefloquine
A

IV artesunate as HIGH parasitaemia

150
Q
  1. Which of the following drugs is used to treat Hepatitis B?
    a. Aciclovir
    b. Oseltamivir
    c. Ribavirin
    d. Tenofovir
    e. Zanamivir
A

Tenofovir used for Hep B, Ribavarin for Hep C + E

151
Q
  1. Infants under 1 year of age shouldn’t be fed honey because of the risk of infection with which organism?
    a. Campylobacter jejuni
    b. Clostridium botulinum
    c. Escherichia coli 0157
    d. Salmonella typhi
    e. Vibrio cholerae
A

Clostridium Botulinum

152
Q

What are anti-ENA antibodies?

A

anti-Ro, La

153
Q

a giril with a sore throat is given penicillin and develops a rash, she is found to have EBV

A

drug reaction NOT allergu

154
Q

woman presents with aches all over and joint stiffness (polymyositis)

A
  1. Anti-Jo1
155
Q
  1. A women with known SLE presents with an exacerbation of her SLE/worsening symptoms, what would you measure?
A

C3 and C4 (C4 is depleted earlier in active disease)

156
Q
  1. A 60 year old lady with recurrent chest infections now has reduced total serum protein and has autoimmune thrombocytopenia
A

Common variable immunodeficiency

157
Q

Young boy with normal B cell numbers and absence of CD8+ and CD4+

A

X-linked SCID

158
Q
  1. Used to treat malignant melanoma, involved in T cell checkpoints
A

Ipilimumab or Nivolumab

159
Q
  1. Used to treat Lymphoma and Rheumatoid arthritis
A

Rituximab

160
Q
  1. Used to prevent transplant rejection, by blocking T cell activation and cell proliferation
A

Tacrolimus or Cyclosporin

161
Q

Antiproliferative agent that worrks on B cell lieage

A

Cyclophosphamide

162
Q
  1. Used to treat severe ankylosing spondylitis not controlled by NSAIDs
A

Etanercept

163
Q
  1. Hypertensive and diabetic, with angioedema
A

Drug reaction to ACE inhibitor

164
Q
  1. Mechanism behind GVHD
A

Donor T cells attack h0ost tissue

165
Q
  1. Mechanism behind antibody mediated rejection
A

B cell mediated

166
Q

Mechanism behind cellular rejection

A

T cell mediated

167
Q

Chimeric antigen receptor T-cell therapy against CD19: what type of haematological malignancy does it target?

A

ALL, some non-Hodgkin lymphoma

168
Q

Ankylosing spondylitis – they’ve tried NSAID and TNF inhibitor, what else can you target?

A

IL-17

169
Q
  1. Which of the following vaccines is a polysaccharide and conjugate vaccine?
A

Pneumococcal vaccine?

170
Q
  1. Woman with periorbital purple rash (heliotrope) and rash on knees, which enzyme is elevated?
A

Creatine kinase (dermatomyositis

171
Q
  1. What Type Hypersensitivity causes serum sickness?
A

Immune complex mediated (type 3)

172
Q
  1. What is the effect of the gene mutation in familial Mediterranean fever?
A

Increased IL-1 production

173
Q

Man, ex-smoker, with history of hypertension and MI, has sudden chest pain which radiates to the neck

A

Left anterior descending artery thrombus

174
Q

A man is found to have a tumour in head of his pancreas that has invaded into portal vein, and there is a palpable mass in the liver

A

Ductal adenocarcinoma (head of pancreas)

175
Q

Kimmelsteil wilson nodules

A

Diabetes nephropathy

176
Q

Which/What cancer is also known as non-specific type?

A

Invasive ductal carcinoma

177
Q

Cancer which has a high, medium and low stage to it

A

Ductal carcinoma in situ

178
Q

A man presents with the symptoms of a UTI and acute onset costovertebral pain. He is found to have pus cells and blood in his urine. What is the most likely cause?

A

Acute pyelonephritis

179
Q

A lady with loin-to-groin pain, haematuria and pain at the end of micturition. What is the most likely cause?

A

Calculi

180
Q

Patient with a history of severe abdominal pain; during cholecystectomy the surgeons noticed grey specks around and on the pancreas

A

Ductal adenocarcinoma

181
Q

Patient had an appendectomy 1 week ago. What would you see in their scar

A

Colllagen

182
Q

On doing a patient’s nephrectomy they notice a mass in the kidney that extends into the the renal vessels and into the perinephric fat

A

Renal cell carcinoma most likely, angiomyolipoma can cause

183
Q

20 year old with a lytic lesion in the diaphysis of his leg, pain alleviated by aspirin

A

osteoid osteoma

184
Q

Man goes deaf with bowed legs

A

Pagets disease

185
Q

Patient with HIV, raised ICP, new onset epilepsy

A

Primary CNS lymphoma

186
Q

Young man with fever, leucocytosis, and a petechial rash after URTI

A

ITP

187
Q

Young guy falls and hits head, two days later loses consciousness

A

Subdural haemorrhage

188
Q

What liver change occurs in diabetic patients

A

Non-alcoholic fatty liver disease or Non-alcoholic steatohepatitis

189
Q

A 15 year old presents with signs of ataxia. Imaging shows a brain tumour extending outwards from cerebellum and invading subarachnoid space. What is the most likely type?

A

Medulloblastoma

190
Q

A patient required an aortic valve replaced 3 months ago. They have now come back a few weeks later with jaundice, Hb-urea (haemoglobinuria), and raised reticulocytes. What is the cause?

A

Valve related haemolysis

191
Q

42 year old woman has a history of intermenstrual bleeding. On examination, a cervical polyp is found. What do you do next?

A

Remove and send for histology

192
Q

What type of section is done for urgent diagnosis during surgery?

A

Intra-operative frozen section

193
Q

Insulinoma. What would you find? Low cortisol, low free fatty acids, low pro-insulin, low glucagon, low c-peptide

A

low free fatty acids

194
Q

50 year old man with Midshaft femur fracture – what would you find at the site of the break/fracture (or rather, which/what type of cancer does he have)?

A

Chondrosarcoma

195
Q

Which trimester does gestational thrombocytopenia occur in? How does this compare to ITP>

A

GTP in third, ITP in first

196
Q

How is therapy in polycythaemia vera monitored?

A

With haematochritM

197
Q

onitoring in patient with AG

A

INR 2-3

198
Q

Monitoring in patient with prosthetic valve and AG

A

INR 2-3

199
Q

What monitoring for continuous unfractionated hepatin?

A

APTT

200
Q

What monitoring for old lady who had cancer having aspiring and clopidogrel?

A

None

201
Q

What is a normal change in pregnancy

A

Gestational thrombocytopenia

202
Q

Cause of neonatal thrombocytopenia

A

Maternal ITP

203
Q

56 y/o lady with SLE, has spherocytes, low Hb, raised bilirubin how do you test for diagnosis?

A

DAT because she has AIHA due to SLE

204
Q

Person who had a DVT many years ago (or recurrent DVTs), presents with recurrent dark bruising, and swelling over the course of 5 years, and now had pain in their leg. What is the possible cause?

A

Post thrombotic syndrome

205
Q

Person who had negative direct coombs test but who has anaemia (low Hb), and jaundice

A

MAHA, valve-associated haemolysis, sickle cell etc.

206
Q
  1. Bortezomib is a proteasome inhibitor, how does this work?
A

Inhibits intracellular protein degradation, build-up and amino acid shortage kills cell

207
Q
  1. What is the mechanism of hyperacute allograft rejection?
A

Pre-formed antibodies attach the graft

208
Q

Lady has newborn baby. Dat +ve, spherocytes seen. Baby jaundiced. Lady is Group A Rhesus negative, Baby is Group O Rhesus positive. Why is baby jaundiced?

A

NOT Hereditary spherocytosis, THIS IS RHESUS DISEASE

209
Q

What is targeted in GvHD?

A

HLA

210
Q

What protein is an-anti thrombotic expressed by vessel walls in their resting state?

A

Thrombomodulin

211
Q

Most characteristic feature of ALL

A

Raised lymphocytes AND lymphadenopathy

212
Q

MACULOPAPULAR RASH MOVES FROM FACE THAT CAN CAUSE ENCEPHALITIS AND PNEUMONITIS

A

Measles

213
Q

VIRUS THAT RESIDES IN PHARYNX AND GIT

A

Polio, have 1:100 ENCEPHALITIS, 1:1000 DESTRUCTION OF MOTOR NEURONS

214
Q

What is herd immunity defined as?

A

THE PROPORTION OF IMMUNE HEALTH PEOPLE NEEDED WITHIN A POPULATION TO STOP A PATHOGEN’S SPREAD: 1- (1/R0)%

215
Q

3 characteristics that can cause a pandemic

A

NOVEL ANTIGENICITY, EFFICIENT REPLICATION IN HUMAN AIRWAY AND EFFICIENT TRANSMISSION BETWEEN HUMANS ARE THE 3 CHARACTERISTICS THAT COULD CAUSE A PANEDEMIC

216
Q

What is most useful test in confirming acute EBC infection 2 weeks after possible exposure in an immunocompetent patient?

A

IGM IS MOST USEFUL TEST IN CONFIRMING ACUTE EBV INFECTION 2 WEEKS AFTER POSSIBLE EXPOSURE IN AN IMMUNOCOMPTENET PATIENT

217
Q

What causes cough, LN enargement and potential to occlude airway?

A

Diphtheria - vaccine preventable

218
Q

What disease causes raised eosinophils?

A

Schistosomiasis

219
Q

Most common cause of cellulitis

A

Staph pyogenesW

220
Q

What is the most likely cause of rash after treatment for infectious mononucleosis?

A

Drug reaction

221
Q

What type of Hepatitis does not lead to chronic hepatitis?

A

Hepatitis A

222
Q

What vaccine can not be given to pregnant women?

A

MMR

223
Q

In a range of scenarios and medical conditions, the platelet count may. be abnormal . Rank these conditions on the likely platelet count. Rank in the order; (1) highest platelet count (5) lowest platelet count.
Essential thrombocythaemia
Rheumatoid arthritis
B Thalassaemia trait
Normal pregnancy
Auto-immune thrombocytopenic purpura

A

Essential thrombocythaemia - This condition is characterized by an abnormally high platelet count due to excessive production of platelets in the bone marrow.
Rheumatoid arthritis - Patients with rheumatoid arthritis may have elevated platelet counts due to chronic inflammation.
Normal pregnancy - Platelet counts can be slightly increased or remain within the normal range during pregnancy.
B Thalassaemia trait - This condition typically does not significantly affect platelet count, so it would generally be within the normal range.
Auto-immune thrombocytopenic purpura - This condition is characterized by a low platelet count due to the body’s immune system attacking and destroying platelets.

224
Q

Regarding the uterine cervix, the process of transformation from normal tissue to a malignant tumour passes through several histologically recognisable stages. Rank the following stages in chronological order starting from earliest (1) to latest (5).
Squamous metaplasia
Mild dysplasia
Moderate dysplasia
Carcinoma in situ
Invasive carcinoma

A

Squamous metaplasia - This is the earliest stage where normal glandular epithelial cells are replaced by squamous cells.
Mild dysplasia - This stage involves mild abnormal changes in the size, shape, and organization of cells.
Moderate dysplasia - This stage involves more pronounced abnormal changes in the cells, but the changes are still confined to the epithelium.
Carcinoma in situ - At this stage, abnormal cells have taken over the entire thickness of the epithelium but have not yet invaded deeper tissues.
Invasive carcinoma - This is the latest stage where cancer cells have invaded beyond the epithelium into surrounding tissues.

225
Q

Healing of an acute myocardial infarction follows an ordered sequence of events, Rank the following events in chronological order starting from earliest (1) to latest (5).
Flocculent densities form within mitochondria
Macrophages. begin to arrive at the area of coagulative necrosis
Neutrophils begin to arrive at the area of coagulative necrosis
Granulation tissue begins to form Collagen is deposited
Forming a fibrous scar

A

Flocculent densities form within mitochondria - This is one of the earliest signs of irreversible injury to myocardial cells, indicating mitochondrial damage and cell death.
Neutrophils begin to arrive at the area of coagulative necrosis - Neutrophils are the first immune cells to arrive at the site of injury, initiating the inflammatory response.
Macrophages begin to arrive at the area of coagulative necrosis - Macrophages arrive after neutrophils and help to clear the debris from the necrotic tissue.
Granulation tissue begins to form - Granulation tissue, which consists of new blood vessels and proliferating fibroblasts, begins to form as part of the healing process.
Collagen is deposited, forming a fibrous scar - Finally, fibroblasts deposit collagen, leading to the formation of a fibrous scar that replaces the damaged

226
Q

Five patients (Patient A to E) each have a a blood gas sample sent to the laboratory, The clinical details of the cases are detailed below, Rank these Cases on the expected pH from (7) lowest pH to (5) highest pH.
Patient D, a 58 year old with very long standing COPD who Is currently quite well
Patient A, a 59 year old with with very long standing COPD who has an acute exacerbation and is feeling breathless
Patient B, is having a panic attack, is hyperventilating and complains of tetany
Patient E, has a cardiac arrest and has blood gases show low pO2 aridand high pCO2
Patient C, a 17 year old with type 1 diabetes who omits his insulin.

A

Patient E - Cardiac arrest with low pO2 and high pCO2, indicating severe respiratory acidosis and possibly metabolic acidosis due to poor perfusion.
Patient C - Type 1 diabetic who omitted insulin, likely to have diabetic ketoacidosis, leading to metabolic acidosis.
Patient A - Long-standing COPD with an acute exacerbation, likely to have acute on chronic respiratory acidosis.
Patient D - Long-standing COPD but currently well, likely to have compensated chronic respiratory acidosis with relatively stable pH.
Patient B - Hyperventilating due to a panic attack, likely to have respiratory alkalosis due to blowing off CO2, leading to an elevated pH.

227
Q

Rank the following diagnoses in order of expected measured serum potassium, with (4) being the highest potassium and (5} being the lowest
Addison’s disease
Pneumonia
Phaeochromocytoma
Cushing’s disease
Conn’s syndrome

A

Addison’s disease - High serum potassium due to adrenal insufficiency leading to decreased aldosterone production, which reduces potassium excretion.
Pneumonia - Generally, serum potassium levels would be normal unless there are complications, making it rank relatively high.
Phaeochromocytoma - Serum potassium levels can be normal or slightly elevated due to catecholamine release, but significant changes are not typical.
Cushing’s disease - Lower serum potassium due to increased cortisol, which can have mineralocorticoid activity leading to increased potassium excretion.
Conn’s syndrome (Primary hyperaldosteronism) - Lowest serum potassium due to excessive aldosterone production leading to increased potassium excretion.

228
Q

A 56 year old female presents with dryness of her eyes, confirmed with ocular staining, and intermittent swelling of her submandibular glands. Rank the a of following investigations in order of specificity for confirmation of her diagnosis, with (1) having the highest specificity and (5) the lowest.
Positive anti-Ro antibody
Positive anti-nuclear antibody
Raised C-reactive protein
Raised immunoglobulin G
Raised neutrophil count

A

Positive anti-Ro antibody - This antibody is highly specific for Sjögren’s syndrome.
Positive anti-nuclear antibody - While not specific to Sjögren’s syndrome, it is commonly found in autoimmune conditions including Sjögren’s.
Raised immunoglobulin G - Elevated IgG can be associated with autoimmune diseases including Sjögren’s, but it is not specific.
Raised C-reactive protein - CRP is a marker of inflammation and is nonspecific, as it can be raised in many inflammatory conditions.
Raised neutrophil count - This is generally indicative of an acute bacterial infection or inflammation and is not specific to Sjögren’s syndrome.

229
Q

A 35 year old man has felt nan- specifically unwell for Sees months and has lost 8kg in weight. He has a fluctuating fever (37- 38.5 degrees) a and bilateral axillary and left inguinal lymphadenopathy. He has no other symptoms and examination is otherwise unremarkabie

Blood test results include:
Hb 102 g/ (130-175) with normochromic normocytic picture
WBC 3.2 with mild lymphopenia
Platelets 162 x 10°/L (150-400)
CRP 34 mg/L (<5)
ESR 86 (<20)
Complement C3 and 4 normal
ANA – negative
ANCA – c-ANCA positive

Renal and bone profiles normal
He’s been investigated for possible infection but all results are negative, including blood film for malaria and elispot for TB.
MPO and PR3 antibodies are pending
Rank the follow in order of likelihood of diagnosis with (1) being most likely and (5) being least
Granulomatosis with polyangiitis (GPA)
Microscopic polyangiitis (MPA)
Chronic granulomatous disease
Lymphome
Systemic lupus erythematosus

A

Granulomatosis with polyangiitis (GPA) - Most likely given the c-ANCA positivity, normochromic normocytic anemia, mild lymphopenia, and elevated inflammatory markers (CRP, ESR). These are characteristic findings for GPA.
Microscopic polyangiitis (MPA) - While MPA can present with similar symptoms, it is more commonly associated with p-ANCA (MPO) positivity rather than c-ANCA.
Lymphoma - The significant weight loss, fluctuating fever, and lymphadenopathy are suggestive of lymphoma, though c-ANCA positivity is not typically associated with it.
Systemic lupus erythematosus (SLE) - The negative ANA makes SLE less likely, as ANA is typically positive in most cases of SLE.
Chronic granulomatous disease - This is a genetic disorder affecting phagocytes, leading to recurrent infections. The patient’s age and presentation are less typical for this condition, and c-ANCA positivity does not align with this diagnosis.

230
Q

A 23 year old man presents with cellulitis having cut himself while skateboarding. A wound swab has grown ‘Streptococcus pyogenes (Group A Streptococcus). List the following antimicrobials which can be used for treatment in order of spectrum of activity with 1 being the narrowest spectrum and 5 being the broadest spectrum agent.
Benzylpenicillin
Amoxicillin
Ceftriaxone
Piperaciilin/tazobactam
Meropenem

A

Benzylpenicillin - This has a very narrow spectrum, primarily targeting Gram-positive organisms, including Streptococcus pyogenes.
Amoxicillin - Slightly broader than benzylpenicillin, it covers a range of Gram-positive and some Gram-negative bacteria.
Ceftriaxone - A third-generation cephalosporin with a broader spectrum than amoxicillin, covering many Gram-positive and Gram-negative organisms.
Piperacillin/tazobactam - This combination has an even broader spectrum, covering a wide range of Gram-positive and Gram-negative bacteria, including Pseudomonas aeruginosa.
Meropenem - This is a broad-spectrum carbapenem antibiotic that covers an extensive range of Gram-positive, Gram-negative, and anaerobic bacteria.

231
Q

A 69 year old man presents with a red, hot swollen knee. He has type 2 diabetes mellitus and osteoarthritis but it is a native joint. In this case of septic. arthritis, rank the causative organisms below in order of likelihood, with (1) being the most likely and 5 the least likely.
Staphylococcus aureus
Streptococcus pyogenes
E coli
Staphylococcus epidermidis
Brucella melitensis

A

Staphylococcus aureus - The most common cause of septic arthritis, especially in patients with diabetes mellitus.
Streptococcus pyogenes - Also a common cause of septic arthritis, though less common than Staphylococcus aureus.
E. coli - More commonly associated with urinary tract infections and gastrointestinal infections but can cause septic arthritis, particularly in elderly or immunocompromised patients.
Staphylococcus epidermidis - Typically associated with prosthetic joint infections rather than native joint infections, but can occasionally cause septic arthritis.
Brucella melitensis - A rare cause of septic arthritis, usually associated with specific risk factors like exposure to livestock or unpasteurized dairy products.

232
Q

A patient is admitted to the ED after severe trauma and blood loss. Without time for blood ABO grouping, he required emergency resus wth RBC and FFP. What blood group should be used?

A

Red Blood Cells (RBCs): O negative (O-) should be used because it is considered the universal donor for RBCs and can be safely given to patients of any blood type.

Fresh Frozen Plasma (FFP): AB should be used because it is considered the universal donor for plasma, meaning it can be safely given to patients of any blood type.

233
Q

Universal donor for RBCs

A

O negative

234
Q

Universal donor for plasma (FFP)

A

AB

235
Q

An elderly man presents with fatigue and recurrent chest infections, The FBC results are reported below, The blood film shows hypogranular hypolobulated neutrophils. What is the most likely cause?

A

Myelodysplastic syndrome

NOTE: Hypogdranular hypolobulated neutrophils - Pelger Huet abnormality

236
Q

A 25 year old Greek woman who is 12 weeks pregnant attends a routine antenatal appointment, the following blood test results are obtained. Based on these results, tp determine if the future health of this fetus may be at risk, what is the first action/investigation required ?

A

Genetic testing of father

237
Q

A 21 year old Syrian woman who has recently moved to the UK from a refugee camp attends a haematology clinic. She has Beta Thalassaemia major and for the last six years has received regular blood transfusions as treatment for anaemia. On examination she has signs of congestive cardiac failure and gonadal failure. What is the most likely explanation for these clinical findings ?

A

Haemosiderosis/Iron overload

238
Q

A 66 year old man with prostate cancer is experiencing fatigue and frank haematuria. The FBC result is presented below. The blood film report notes the presence of, myelocytes, tear drop poikilocytes and nucleated red blood cells. What is the most likely explanation for the anaemia

A

Myelofibrosis - tear drop poikilocytes

239
Q

What is the effect of male sex on the risk of venous thrombosis recurrence?

A

Increased, male have higher chance of recurrence

FEmales have higher chance of first epidose

240
Q

Folic acid supplementation taken periconceptually and in early pregnancy is important to prevent what condition?

A

Neural tube defects

241
Q

A 52 year old woman who is a vegan, is taking carbimazole therapy as treatment for hyperthyroidism, She develops fatigue and easy bruising. The FBC results are reported below. What is the most likely diagnosis?

A

Agranulocytosis - secondary to meds

OR aplastic anaemia

242
Q

Which leucocyte actively participates in an acute inflammatory reaction, contains myeloperoxidase within its primary granules and alkaline phosphatase in it secondary granules?

A

Neutrophils

243
Q

A plain chest X-ray of a 47 year old male reveals a 2.5 cm “coin lesion” in the upper lobe of his left lung. The lesion is removed. The report describes a clear cell carcinoma. What is the most likely primary site of origin for this carcinoma.

A

kidney

244
Q

What is the most common histologic type of carcinoma involving the oral cavity?

A

Squamous cell carcinoma

245
Q

A patient with Sjogren’s syndrome has a salivary gland biopsy. What cells are the most common infiltrating cell type likely to be seen?

A

Lymphocytes - likely to be in the build up to a parotid gland marginal zone lymphoma

246
Q

A 42 year old male presents in renal failure, His kidneys are grossly enlarged by multiple large, thin-walled cysts. Some of the cysts contain blood clots. What is the likely diagnosis?

A

PCKD

247
Q

A 42 year old male has fever, night sweats and haemoptysis. A transbronchial lung biopsy shows granulomas with necrosis. What is the most likely cause?

A

TB

248
Q

The combination of hemangioblastomas in the cerebellum and retina, multiple and bilateral renal cell carcinomas, and cysts of the pancreas and kidneys is characteristic of what neurocutaneous syndrome ?

A

Von-Hippel Lindau

249
Q

A 39 year old male farmer from Southern Africa has been exposed to mouldy grain during his working life. He has worsening abdominal pain and jaundice. There is a large mass in the right lobe of his liver. Biopsy of the mass reveals hepatocellular carcinoma, What extrinsic agent is likely to have played a role in the development of this tumour?

A

Alfatoxin

250
Q

A patient with multiple endocrine neoplasia type 2 has had a thyroidectomy for medullary thyroid carcinoma but now presents with hypertension and. palpitations. What is the likely diagnosis?

A

Phaeochromocytoma

251
Q

A 24 year old patient with known type 1 diabetes presents having missed her insulin, and is now vomiting with Kussmaul’s respiration. Predict the likely bicarbonate concentration in this patient. (bicarbonate reference range 22-29 mmol/L)

A

Low - around 18

252
Q

A 32 year old female physiotherapist is overweight with excess facial hair and also experiences recurrent acne. Over the past nine months her periods have been infrequent. Testosterone 3.7 nmol/L (0.22.9), What is the likely diagnosis?

A

PCOS

253
Q

A neonate presents with a salt losing crisis, and is thought to have congenital adrenal hyperplasia. What is the commonest enzyme deficiency that causes this? Name the enzyme

A

21-alpha hydroxylase

254
Q

A 24 year old woman with known type 1 diabetes accidentally missed an insulin dose. She presents with vomiting and Kussmaul’s respiration. Predict in kPa the likely pCO, reading that you will find on the Arterial Blood Gas sample. pCO, normal range (4.6-6.4 kPa)

A

Low -3ish

255
Q

30yo man comes to ED having taken a drug overdose. ABG shows normal pH with really low pCO2. What drug is he likely to have taken?

A

Salicylates

256
Q

Elevated ALP in the presence of no Sx

A

Paget’s

257
Q

A monoclonal antibody specific for receptor activator nuclear factor kappa-b ligand (RANK ligand) is effective in management of which condition?

A

Osteoporosis - Denosumab

258
Q

Which cytokine is key in promoting the development and terminal differentiation of eosinophils?

A

IL-5

259
Q

A 49 year old male presents with haemoptysis and acute renal failure. Ix confirm pulmonary haemorrhage and glomerulonephritis. The renal biopsy is sent for immunofluorescence and this shows a linear deposition of antibody along the glomerular basement membrane. The effector mechanism for immunopathology in this condition is typical of which type of Gel and Coombs hypersensitivity reaction?

A

T2 - Goodpasture’s

260
Q

A 7 month old male child, presents with pneumococcal pneumonia. He has a history of cellulitis and recurrent otitis media. A full blood count showed neutrophilia. Flow cytometry confirmed presence of CD19+ B cells and CD3+ T cells, including both CD8+ and CD4+ T cells. Serum immunoglobulin levels are shown below:
IgA <0.1 (0-0.83)
IgG 0.02 (2.3-14)
IgM 1.84 (0-1.45)
Mutations in which gene most likely explains this presentation?

A

CD40L on CD4+ cells

261
Q

Antibodies bind te which cell type during the effector phase of antibody mediated rejection of solid organ allografts?

A

endothelial cells

262
Q

A young man presents with episodes of fever, pleurisy and peritonitis. Investigations: confirm a mutation of the MEFV gene encoding pyrin. Which drug inhibiting neutrophil function is regarded as the agent of choice for prophylaxis against acute febrile illness and prevention of amyloidosis?

A

Colchicine

263
Q

A 25 year old man has a 10 year history of low back pain and stiffness, with symptoms being worse at night and in the early mornings. An MRI scan has shown bone marrow oedema at the sacroiliac joints. Blood tests confirm an acute phase response with C- reactive protein 18 mg/L (<5). He has shown a partial response to NSAIDS. Which cytokine could be targeted to improve disease control.

A

TNF-alpha

264
Q

What is the maximum number of HLA class mismatches that may occur if a parent acts as a kidney donor for a child?

A

3

The maximum number of HLA class mismatches that may occur when a parent acts as a kidney donor for a child is usually 3 mismatches. This is because each person inherits one set of HLA genes from each parent, resulting in a potential maximum of 6 mismatches (as there are typically 6 HLA antigens tested for kidney transplantation: HLA-A, -B, -C, -DR, -DQ, and -DP). However, since the parent and child share half of their HLA antigens due to genetic inheritance, the maximum number of mismatches would be half of the potential 6, which is 3 mismatches.

265
Q

Patient who is blood group O is on the waiting list for kidney transplant. A potential donor should have which blood group?

A

Any

266
Q

What species of Coagulase Negative Staphylococcus is a common cause of lower urinary tract infection in young women?

A

Staph Saprophyticus

267
Q

A 67-year-old woman presents with a chronic cough, haemoptysis and fevers particularly at night. She states she has lost 10 kg of weight over the past 3 months and her chest X-ray shows left upper zone shadowing. Her son reports she has a history of excess alcohol consumption. What is the most likely causative organism?

A

TB

268
Q

What double stranded DNA virus is associated with nephritis in renal transplant recipients on immunosuppresive therapy?

A

BK virus

269
Q

A 35 year old person who injects drugs (PWID) is admitted with fever, shortness of breath and haemoptysis. Her chest x-ray has been reported as showing multiple pulmonary infiltrates compatible with pulmonary emboli. Which ONE microbiology investigation should you send before commencing empiric abx?

A

Blood cultures

270
Q

A 32 year old man presents with fever, abdominal pain and constipation. He has recently returned from Pakistan. He undertook no vaccinations prior to travel. Blood cultures taken on admission have grown non-lactose fermenting Gram negative bacilli in both bottles. What is the most likely causative organism?

A

Salmonella Typhii

271
Q

A 32 year old woman has been diagnosed with a superficial infection of a caesarean section wound. The preliminary report from a wound swab states: Staphylococcus Aureus isolated with susceptibilities to follow. The pre-operative MRSA screen was negative and there are no known any drug allergies: She is breast feeding her new daughter. What narrow Spectrum antimicrobial would you prescribe to treat his infection?

A

Flucloxacillin

272
Q

Which helminth is capable of autoinfection in humans via the following life cycle?
Infective filariform larvae penetrate intact skin and migrate to the small intestine where they became adults. Eggs hatch into rhabaitifom larvae which mature into filariform larvae which can autoinfect via perianal skin.

A

Strongyloides Stercoralis

273
Q

A 26 year old woman presents with fever, headache and confusion. She has an HIV infection and has refused to take antiretroviral therapy for the past 4 years. What rapid test should you ask the microbiology laboratory to perform on her serum sample to confirm the likely diagnosis?

A

Cryptococcal antigen test

274
Q

A 2-day old neonate develops meningitis. The microbio laboratory telephone to report that a Gram stain of her CSF shows Gram- negative bacilli. What is the most likely causative organism?

A

E.Coli

275
Q

Urinary antigen tests are useful in the diagnosis of community acquired pneumonia. State one organism that the commonly available tests can detect?

A

Legionella urinary antigen

276
Q

Which vaccine preventable disease presents initially with mild fever, swollen neck glands, anorexia, malaise. and cough. After 2-3 days a membrane of dead cells forms in the throat, tonsils, larynx or nose which may narrow or occlude the airway leading to respiratory distress?

A

Diphteria

277
Q

What Hepatitis Virus typically causes a brief, self limiting infection but is associated with more severe disease and mortality in pregnant women and can cause chronic infection in immunocompromised patients?

A

Hep E

NOTE: Hep E can only cause chronic infection in immunocompromised patients

278
Q

A 36 year old woman presents. with a headache and hemiparesis. She also has a six week history of bruising. and epistaxis. On examination there is a widespread petechial rash. Laboratory results are presented below. Bone marrow aspirate morphology reports an excess of promyelocytes. Genetic analysis of the malignant cells is expected to reveal which aberration?
BCR/ABLI fusion gene.
Chromosomal translocation involving c-myc proto oncogene
JAK2 V617F mutation
PML/RARA fusion gene
Trisomy 21

A

PML/RARA fusion gene - presents with DIC and massive haemorrhage

279
Q

A 35 year old woman attends an antenatal clinic at 16 week gestation. She is blood group A RhD. negative, A previous pregnancy was affected by severe Rhesus D haemolytic disease of the newborn. Free foetal DNA analysis determines the foetal genotype as RhD negative. Which management approach is correct?

A

No further action required

280
Q

A patient who is known to have chronic lymphocytic leukaemia presents with sudden worsening of fatigue and is found to have had a fall in Hb. His FBC results are reported below. His blood film shows lymphocytosis, smear cells, spherocytes and polychromatic macrocytes. Which is the most likely cause of his fall in Hb?

A

AIHA - warm haemolytic anaemia due to CLL cause spherocytes

281
Q

In optimal circumstances, for a 70kg male, transfusion of 1unit RBC and 1 unit of platelet gives a predictable increment in the Hb and platelet count. Which statement is correct?
Expected increment, Haemoglobin 10 g/L and Platelets 400 x 10^9/L.
Expected increment, Haemoglobin 20 g/L and Platelets 10 x 10^9/L.
Expected increment, Haemoglobin 50 g/L and Platelets 35 x 10^9/L
Expected increment, Haemoglobin 50 g/l and Platelets 100 x 10^9/L.
Expected increment, Haemaglobin 10 g/L and Platelets 35 x 10^9/L.

A

Expected increment, Haemaglobin 10 g/L and Platelets 35 x 10^9/L.

NOTE: In optimal circumstances, the transfusion of 1 unit of red blood cells (RBCs) is expected to increase the hemoglobin level by approximately 10 g/L in a 70 kg male. Similarly, the transfusion of 1 unit of platelets is expected to increase the platelet count by approximately 30-60 x 10^9/L.

Therefore, if 1 unit of RBCs and 1 unit of platelets are transfused, the expected increment in hemoglobin would be around 10 g/L, and the expected increment in platelets would be around 30-60 x 10^9/L. Choosing the closest option, the statement “Expected increment, Hemoglobin 10 g/L and Platelets 35 x 10^9/L” is the most accurate approximation of the expected increments.

282
Q

In allogeneic haemopoietic stem cell transplantation for acute myeloid leukaemia. Removing all donor derived T lymphocytes (T-cell depletion) from the donor cell collection prior to infusion into the recipient has an impact on both, the incidence of acute graft, versus host disease (aGVHD) and the rate of leukaemia relapse. Which statement is true?
Abolishes aGHVD risk and removes risk of leukaemia relapse .
Increases aGVHD risk and increases leukaemia relapse rate.
Increases aGVHD risk and reduces leukaemia relapse rate.
Reduces aGVHD risk and reduces leukaemia relapse rate.
Reduces aGVHD risk and increases leukaemia relapse rate.

A

Reduces aGVHD risk and increases leukaemia relapse rate.

NOTE: T-cell depletion in allogeneic hematopoietic stem cell transplantation (HSCT) reduces the risk of acute graft-versus-host disease (aGVHD) by removing donor T cells that could attack the recipient’s tissues. However, T-cell depletion also removes the graft-versus-leukemia (GVL) effect, where donor T cells recognize and attack residual leukemia cells in the recipient. As a result, the rate of leukemia relapse may increase with T-cell depletion.

283
Q

A 30 year old woman with known ITP gives birth to a healthy full term baby. Which is the correct haematological management of the newborn baby?
Check daily bilirubin levels for a minimum of five days
Check daily FBC for a minimum of five days.
Check FBC on cord blood sample and if normal no further action required
Check INK on cord blood sample and administer Vitamin K
Perform DAT (direct anti-globulin test) on both maternal and neonatal red blood cells.

A

Check FBC on cord blood sample and if normal no further action required.

Immune thrombocytopenic purpura (ITP) is a condition characterized by a low platelet count due to the destruction of platelets by the immune system. While maternal ITP can lead to low platelet counts in the newborn due to passive transfer of maternal antibodies, in most cases, these platelet counts are transient and resolve without intervention.

Checking a full blood count (FBC) on cord blood sample allows for assessment of the newborn’s platelet count. If the platelet count is within normal range, no further action is usually required. However, if the platelet count is low, further evaluation and management may be necessary.

284
Q

Renal transplant recipients on long term immunosuppression have an increased risk of which lymphoid cancer?
Cutaneous T cell lymphoma.
EBV associated Diffuse large B cell lymphoma (DLBCL)
Enteropathy associated T cell lymphoma
H.Pylori associated gastric marginal zone lymphoma
HTLV1 associated Adult T cell lymphoma (ATLL)

A

EBV associated Diffuse large B cell lymphoma (DLBCL)

285
Q

In B cell Chronic lymphocytic leukaemia (8 CLL), an orally active medication that is an inhibitor of which tyrosine kinase is likely to be an effective therapy?
ABL (Abelson) tyrosine kinase (Imatinib)
Bruton tyrosine kinase (Ibrutinib)
Human epidermal growth factor 2 (HER2) (Lapatinib)
Janus Kinase JAK) (Ruxolitini)
Rapidly accelerating fibrosarcoma (B RAF) (Vemurafenib)

A

Bruton tyrosine kinase (Ibrutinib)

286
Q

Which of the following is associated with disseminated intravascular coagulation?
Decrease in neutrophil count
Decrease in reticulocyte count
Increased plasma fibrinogen concentration
Increased platelet count
Prolonged thrombin time

A

Prolonged thrombin time

287
Q

A 55 year old man is being discharged from hospital on long term anticoagulation with Rivaroxaban a direct acting anticoagulant. Which is the correct approach for routine monitoring of his anticoagulation?
anti Xa assay
APTT (Activated Partial Thromboplastin Time)
D-Dimers assay
INR (International normalised ratio)
Routine laboratory
Monitoring not required

A

Monitoring not required

288
Q

Which of these tumours is most likely to be associated with Pseudomyxoma peritonei?
Clear cell tumour of the kidney
Cystic tumour of the gallbladder
Mucinous tumour of the ovary
Serous tumour of the ovary
Smooth-muscle tumour of the uterus

A

Mucinous tumours of the ovary

289
Q

A 26 year old person had silicone breast implants inserted 2 years ago. They are now asymmetrical, and one has become much harder than before. The leaking implants are removed and sent to pathology. Which inflammatory cell is most prominent on histological examination?
Eosinophil
Macrophage giant cell
Mast cell
Neutrophil

A

Macrophage giant cell

290
Q

A 42 year old male with alcoholism and portal HTN presents with haematemesis and severe hypotension. He dies during attempted resuscitation. At post- mortem examination of the lower oesophagus, which is the most likely finding?
Acute inflammation
Colummar epithelium (Barrett metaplasia)
Decreased ganglion cells in the myenteric plexus
Dilated blood vessels in the submucosa
Mucosal outpouchings (diverticulae)

A

Dilated blood vessels in the submucosa, also known as esophageal varices.

Portal hypertension, a complication often associated with chronic liver disease, including alcohol-related liver disease, can lead to increased pressure in the portal vein system. This increased pressure can cause the development of collateral circulation, leading to the formation of esophageal varices, particularly in the lower esophagus.

291
Q

A 27 year old African American female is unwell with non specific symptoms including fever and malaise. A chest x -ray SHOWS enlarged hilar lymph nodes. Serum calcium level is elevated, A hilar lymph node is biopsied. Which histological feature is most likely to be found?
Caseating granulomatous inflammation
Dense, granular, PAS-positive, eosinophilic material
Markedly enlarged epithelial cells with intranuclear inclusions
Non-caseating granulomatous inflammation
Numerous neutrophils with fibrin deposition

A

Non-caseating granulomatous inflammation - sarcoidosis

292
Q

A 57 year old male presents with anaemia and fatigue. Endoscopy identifies a bleeding tumour in the sigmoid colon. The tumour is completely resected, and no metastatic disease found. Which biochemical marker would be most useful to monitor for possible recurrence?
Carcinoembryonic antigen (CEA)
Chioroacetate esterase (CAE}
Human chorionic gonadotropin (hCG)
Prostate-specific antigen (PSA)
a fetoprotein (AFP)

A

CEA

293
Q

A 7 year old male has acute onset severe 2 testicular pain. He is diagnosed with testicular torsion and the testis is removed surgically. Histology shows haemorrhagic testicular infarction. Which process causes the haemorrhagic findings?
There has been complete arterial occlusion
There has been venous occlusion
There is a collateral blood supply the the testis
There is a dual blood supply to the testis
There is septic infarction

A

Complete arterial occlusion.

Testicular torsion involves the twisting of the spermatic cord, which contains the testicular artery and veins. This twisting results in the compression and subsequent occlusion of the testicular artery, leading to ischemia (lack of blood supply) of the testicle. Without adequate blood flow, the testicular tissue becomes infarcted, resulting in tissue necrosis and hemorrhage.

294
Q

A 59 year old male is found to have anaemia due to blood loss from a tumour arising in the colon. A biopsy is taken from the mass and the pathologists conclusion is invasive adenocarcinoma. Which descriptive phrase is most likely to have been used in the pathology report?
A disorganized mass of cells forming glandular structures
A disorganized mass of cells forming keratin _
A disorganised mass of proliferating fibroblasts and blood vessels
A uniform proliferation of fibrous tissue
A uniform proliferation of glandular structures

A

A disorganized mass of cells forming glandular structures.

Adenocarcinoma is a type of cancer that originates in glandular cells, and it is characterized by the abnormal proliferation of these cells, which form disorganized glandular structures.

295
Q

A 27 year old woman in her fifteenth week of pregnancy presents with uterine bleeding. She is also passing watery fluid and tissue PV. Her uterus is much larger than predicted by her gestational dates. Ultrasound scanning shows cystic grape-like structures that do not penetrate the uterine wall. No foetal parts are found. Which is the most likely diagnosis?
Choriocarcinoma
Complete hydatidiform mole
Invasive mole
Partial hydatidiform mole
Placental site trophoblastic tumour

A

Complete hydatidiform mole.

Complete hydatidiform mole, also known as complete molar pregnancy, is characterized by the abnormal proliferation of trophoblastic tissue within the uterus, leading to the formation of grape-like cystic structures (vesicles) that do not contain fetal parts. These cystic structures arise from the abnormal fertilization of an egg without a nucleus by either two sperm (complete mole) or one sperm that duplicates its genetic material (diandric triploidy).

296
Q

A 42 year old male with alcoholism. presents with signs of alcoholic hepatitis, A liver biopsy shows cytoplasmic regular eosinophilic hyaline bodies in the hepatocytes. From which substance are these inclusions formed?
Basement membrane material
Excess plasma proteins
Immunoglobulin
Lipofuscin
Pre-keratin intermediate filaments

A

Mallory-Denk bodies. These structures are composed of pre-keratin intermediate filaments.

297
Q

A 75 year old male presents with dysphagia and is feu to have a 4m mass in the middle third of the oesophagus which invades into the middle third of the oesophagus which invades into the adjacent lung. Biopsy from this mass would mass likely show which histological feature?
A mass composed of benign cartilage
A mass composed of benign smooth-muscle cells
Infiltrating groups of malignant cells forming glandular structures
Infiltrating malignant single calls having intracellular mucin
Infiltrating sheets of malignant cells forming keratin

A

Infiltrating sheets of malignant cells forming keratin

Given the location in the middle third of the esophagus, squamous cell carcinoma is indeed a more likely possibility.

298
Q

A 42 year old male cigarette smoker presents with urinary frequency and haematuria. Cystoscopy finds an ‘exophytic fronded lesion’ which is biopsied. Histologic examination shows fibrovascular cores covered by atypical cells. No invasive malignancy is seen. No glands are seen, and no keratin production is found. Dx for this bladder tumour?
Adenocarcinoma
Inverted papilloma
Papillary urothelial carcinoma
Squamous cell carcinoma
Urothelial carcinoma in situ

A

Urothelial carcinoma in situ (CIS).

Urothelial carcinoma in situ (CIS), also known as carcinoma in situ, is a non-invasive high-grade lesion characterized by atypical urothelial cells that involve the full thickness of the urothelium without invasion into the lamina propria or beyond.

299
Q

Which of the following electrolytes is raised in the plasma following haemolysis?
Bicarbonate
Calcium.
Chloride
Potassium
Sodium

A

Potassium

300
Q

A 64-year-old woman. with known type 2 diabetes, peripheral vascular disease and ischaemic heart disease is started on ramipril for hypertension. Two days later she is admitted with pruritus and pulmonary oedema and has the following results: Dx?
Glomerulonephritis
IgA nephropathy
Multiple myeloma
Renal artery stenosis
Rhabdomyalysis

A

Renal artery stenosis

301
Q

A 40 year old woman has 6 months of urinary frequency, passing urine every hour even at night. She has bipolar affective disorder d and take lithium carbonate.
Ix shows:
Thyroid stimulating hormone - 5 (0.3-4.2)
Free T4 - 10.2 (9-25)
Sodium - 147 (135-146)
Potassium - 3.5 (3.5-5)
Ureal - 8.8 (2.5-7.8)
Creatinine 107 (60- 120)
Fasting glucose 5.8 (3.0-6.0)

Which is the most likely cause of her symptoms?
Diabetes insipidus
Diabetes mellitus
Hypothyroidism
Psychagenic polydipsia
Syndrome of inappropriate antidiuretic hormone secretion (SIADH)

A

Diabetes Insipidus - Nephrogenic - treated with thiazide diuretics

302
Q

Which of the following transports cholesterol to the liver and steroidogenic organs (adrenals, ovary, testes}?
HDL
LDL
Lipoprotein lipase
Triglyceride
VLDL

A

HDL

303
Q

A25 year old woman presents with a purple rash around the eyes and a raised red rash on her elbows and knees. Which enzyme would you expect to be raised?
Alkaline Phosphatase
Amylase
Aspartate Amino-transferase
Creatine kinase
Glucose-6-phasphate dehydrogenase

A

Creatine Kinase

304
Q

Which of the following stimulate the release of prolactin from the pituitary gland?
Corticotrophin releasing hormone (CRH)
Dopamine
Gonadotrophin releasing hormone (GnRH)
Growth hormone releasing hormone {GHRH}
Thyrotropin releasing hormone (TRH)

A

TRH

305
Q

In a disease such a phenylketonuria, which term describes the number of true negatives divided by the total number of people that do not have the disease?
Negative predictive value
Positive predictive value
Sensitivity
Specificity
z-score

A

pecificity.

Specificity measures the proportion of individuals without the disease who test negative on a diagnostic test. In the context of phenylketonuria (PKU), which is a metabolic disorder, specificity would indicate the proportion of individuals who do not have PKU but have negative test results for the condition.

Negative predictive value (NPV) refers to the probability that a person with a negative test result truly does not have the disease. Positive predictive value (PPV) refers to the probability that a person with a positive test result truly has the disease. Sensitivity measures the proportion of individuals with the disease who test positive on a diagnostic test. Z-score is a statistical measure used in hypothesis testing and represents the number of standard deviations a data point is from the mean of the population.

306
Q

Which of the following is a feature of both Multiple Endocrine Neoplasia type 1 and Multiple Endocrine Neoplasia type 2a?
Acromegaly
Gastrinoma
Medullary thyroid carcinoma
Phaeochromocytoma
Primary hyperparathyroidism

A

Primary hyperparathyroidism

307
Q

A 4 year old child presents with fever, weight loss, malaise and abdo pain. CT abdomen shows hepatosplenomegaly and retroperitoneal lymphadenopathy. Mycobacterium aviurm-intracellulare is grown from blood cultures. Which immune deficiency is most likely to explain the presentation?
Bare lymphocyte syndrome type 2
Chronic granulomatous disease
IgA deficiency
Interferon gamma receptor deficiency
Mannose binding lectin deficiency

A

Interferon gamma receptor deficiency.

Interferon gamma receptor deficiency leads to impaired cellular immune response, particularly involving macrophages, which are crucial for the defense against intracellular pathogens such as mycobacteria. As a result, individuals with interferon gamma receptor deficiency are susceptible to severe and disseminated infections with mycobacteria, including Mycobacterium avium-intracellulare.

308
Q

Alum is-effective as an adjuvant for vaccination. Which of the following best describes its mechanism of action?
Enhancement of CD4+ T cell response
Enhancement of the innate immune response
Promotion of B cell differentiation
Suppression of CD& T ceil response
Suppression of the T reg response

A

Enhancement of the innate immune response

Alum adjuvant works by promoting the activation of antigen-presenting cells (such as dendritic cells) and inducing the release of pro-inflammatory cytokines, such as interleukin-1 (IL-1) and interleukin-18 (IL-18). This leads to the recruitment of immune cells to the site of injection and enhances the overall immune response to the vaccine antigen.

309
Q

Nivolumab is a monoclonal antibody specific for programmed death 1 (PD-1) and may be used as part of management for metastatic melanoma. Its principle mechanism of action is to activate which cell type?
B-lymphocytes
Endothelial cells
Melanocytes
Neutrophils
T-lymphocytes

A

T-lymphocytes.

PD-1 is a receptor found on the surface of T cells, among other immune cells. When PD-1 interacts with its ligands PD-L1 or PD-L2, it inhibits T cell activation and proliferation, contributing to immune tolerance and evasion by tumor cells.

310
Q

Mutations in FOXp3 cause IPEX by directly affecting the development of which cell type?
Dendritic cells
Neutrophils
Pre-B cells
T-follicular helper cells
T-regulatory cells.

A

Mutations in the FOXP3 gene cause IPEX (Immune dysregulation, Polyendocrinopathy, Enteropathy, X-linked syndrome) by directly affecting the development of T-regulatory cells (Tregs).

FOXP3 is a transcription factor that plays a critical role in the development and function of Tregs, which are a subset of T cells involved in maintaining immune tolerance and suppressing excessive immune responses.

311
Q

A 45 year old woman has rheumatoid arthritis and is taking methotrexate and adalimumab (ab specific for TNFa) to manage her condition. Which vaccination is contraindicated?
Hepatitis B vaccine
pneumococcal polysaccharide vaccine
Quadrivalent influenza vaccine
Tetanus vaccine
Yellow fever vaccine

A

Yellow fever vaccine would be contraindicated in this patient.

The yellow fever vaccine is a live attenuated vaccine, and its use is generally contraindicated in individuals with immunosuppression due to the risk of developing yellow fever vaccine-associated viscerotropic disease (YEL-AVD) or yellow fever vaccine-associated neurotropic disease (YEL-AND), which can be severe or fatal in immunocompromised individuals.

312
Q

Secukinumab, an antibody specific for interleukin (IL) 17A, may be used as part of management in which one of the following diseases?
Eosinophilic Granulomatosis with Polyangiitis
Giant cell arteritis
Metastatic malignant melanoma
Psoriatic arthritis
Rheumatoid arthritis

A

Psoriatic arthritis.

Psoriatic arthritis is a chronic inflammatory arthritis that occurs in some individuals with psoriasis, a skin condition characterized by red, scaly patches. IL-17A is known to play a key role in the pathogenesis of psoriatic arthritis by promoting inflammation and tissue damage in the joints.

313
Q

A 45 year old man has a long history of cutaneous psoriasis and has developed features of a spondyloarthritis, with sacroiliitis and knee effusions. He has been extensively investigated, including with blood tests for anti-nuclear antibody, anti- dsDNA antibody, rheumatoid factor, anti-cyclic citrullinated peptide antibody. What would you expect results of these serological tests to show?
Negative for all
Positive anti CCP ab
Positive anti-dsDNA ab
Positive anti-nuclear ab
Positive rheumatoid factor

A

Negative for all

314
Q

Which biologic disease modifying anti- rheumatic drug (DMARD) targeting a specific cytokine pathway would be expected to show efficacy in RA but not psoriatic arthritis?
Adalirnumab (anti-tumour necrosis factor alpha antibody)
Etanercept (tumour necrosis factor receptor - Ig fusion protein
Secukinurnab (anti-interleukin 17antibody)
Tocilizumahb (anti-interleukin 6 receptor antibody)
Ustekinumab (Ab specific for p40 of interleukin 12 and interleukin 23)

A

Tocilizumab, which is an anti-interleukin 6 receptor antibody.

Tocilizumab targets the interleukin-6 (IL-6) pathway, which plays a significant role in the pathogenesis of RA by promoting inflammation and joint damage. Therefore, blocking IL-6 signaling with tocilizumab has shown efficacy in treating RA.

However, PsA is driven by different cytokine pathways, including tumor necrosis factor alpha (TNF-alpha) and interleukin-17 (IL-17), rather than IL-6. Therefore, drugs targeting the IL-6 pathway, such as tocilizumab, may not be as effective in treating PsA compared to RA.

NOTE: Adalimumab and Etanercept target tumor necrosis factor alpha (TNF-alpha) and are effective in both RA and PsA.
Secukinumab targets interleukin 17A (IL-17A) and is effective in treating PsA but not RA.
Ustekinumab targets the p40 subunit shared by interleukin-12 (IL-12) and interleukin-23 (IL-23) and is effective in treating PsA but not RA.

315
Q

Cigarette smoking is associated with development of rheumatoid arthritis. Which is the most likely mechanism underpinning this association?
Nicotine acts as.an immune checkpoint inhibitor
Nictoine as an inhibitor of T reg
Peptide from nicotine binds to the shared epitope of HLADR1 and DR4
Smoking increases citrullination of proteins in the lung
Smoking results in high levels of hydrogen cyanide which stimulated production of RF

A

Smoking increases citrullination of proteins in the lung

Citrullination is a post-translational modification where the amino acid arginine is converted to citrulline. This process can lead to the generation of autoantigens that trigger an immune response, including the production of anti-citrullinated protein antibodies (ACPAs). These antibodies are highly specific for RA and are associated with more severe disease manifestations.

316
Q

The following antimicrobials can be used for the treatment of community acquired pneumonia. Which has activity against Mycoplasma pneumoniae?
Amoxicillin
Ceftriaxone
Co-amoxiclav
Doxycycline
Vancomycin

A

Doxycycline - onlt one with activity against mycoplasma

317
Q

A 76 year old man with a long term indwelling urinary catheter which was changed earlier in the day becomes unwell. He is pyrexial and has been having rigors. He is tachycardic heart rate of 123 bpm, and hypotensive blood pressure 94/62 mmHg. He is known to be colonised with an ESBL producing E.coli. Which antimicrobial would be the mast appropriate empiric therapy?
Aztreonam
Ceftriaxone
Meropenem
Piperacillin/tazobactam
Trimethoprim

A

Meropenem

318
Q

A 27 year old accountant presents with a 3 day history of abdominal cramps and diarrhoea. She is afebrile, is eating and drinking well and feels her symptoms have started to resolve. She is currently working from home and one week ago ate a reheated take away chicken curry. Which treatment is indicated.
Amoxicillin
Azithromycin
Ceftriaxone
Ciprofloxacin
No antimicrobial therapy indicated, supportive only

A

No antimicrobial therapy indicated, supportive only - likely just gastroenteritis

319
Q

A 4 year old girl presents with pharyngitis. A throat swab has isolated Streptococcus pyogenes (Group A Streptococcus). Her father states she had an anaphylactic reaction to penicillin 18 months ago. While awaiting susceptibilities, which antimicrobial would be most appropriate initial therapy?
Ceftriaxone
Ciprofloxacin
Clarithromycin
Doxycycline
Piperacillin/tazobactam

A

Clarithromycin

Clarithromycin is a macrolide antibiotic that is effective against Streptococcus pyogenes and is an alternative to penicillin for the treatment of streptococcal pharyngitis. It does not contain a beta-lactam ring and is not associated with cross-reactivity with penicillin in patients with penicillin allergy.

320
Q

Which component of SARS CoV-2 is the antigenic target for the vaccines currently used in the UK?
Envelope (E) protein
Membrane (iM) protein
Nucleocapsid (N)protein
Spike (S) protein
Viral RNA

A

Spike protein

321
Q

Which agent is an Interleukin 6 (IL-6) inhibitor, indicated for treatment of severe COVID-19 infection with hypoxia?
Anakinra
Nafamostat
Palivizumab
Ruxalitinib
Tocilizumab

A

Tocilizumab

Tocilizumab is a monoclonal antibody that specifically targets the interleukin-6 (IL-6) receptor, thereby inhibiting the pro-inflammatory effects of IL-6. It has been used in the treatment of severe COVID-19 cases, particularly in patients with cytokine release syndrome (CRS) or hyperinflammatory states associated with the disease.

322
Q

Streptococcus pneumoniae is resistant to penicillin, by which mechanism?
Alteration of the target that binds the antimicrobial
Bypass of the antimicrobial susceptible step in metabolism
Enhanced efflux of the antimicrobial
Enzymatic inactivation of the antimicrobial
Impaired uptake of the antimicrobial into the bacterial cell

A

Alteration of the target that binds the antimicrobial.

The mechanism involves the acquisition of mutations in the genes encoding penicillin-binding proteins (PBPs), which are the targets of penicillin. These mutations lead to changes in the structure of PBPs, reducing the affinity of the bacterial cell wall for penicillin and rendering the antibiotic less effective in inhibiting cell wall synthesis. As a result, the bacteria become less susceptible to the effects of penicillin.

323
Q

Which antiviral agent is used for the treatment of Influenza A?
Cidofovir
Foscarnet
Ribavirin
Tenofovir
Zanamivir

A

Zanamivir

Zanamivir is a neuraminidase inhibitor that is effective against both Influenza A and Influenza B viruses. It works by inhibiting the activity of the viral neuraminidase enzyme, which is essential for the release of newly formed virus particles from infected cells and the spread of the virus within the respiratory tract. By inhibiting neuraminidase, zanamivir reduces viral replication and helps to alleviate symptoms of influenza.

324
Q

Blood cultures taken from a 57 year old patient on renal dialysis have grown a glycopeptide resistant Enterococcus faecium. Further susceptibilities are to follow. Which antimicrobial is an appropriate choice for initial treatment of this infection?
Co-trimoxazole
Linezolid
Meropenem
Taicopianin
Vancomycin

A

Linezolid

Linezolid is an oxazolidinone antibiotic with activity against Enterococcus faecium, including strains that are resistant to glycopeptides such as vancomycin. It inhibits bacterial protein synthesis by binding to the 23S ribosomal RNA of the 50S subunit, thereby preventing the formation of the initiation complex for protein synthesis. Linezolid is commonly used for the treatment of infections caused by vancomycin-resistant Enterococcus (VRE).

325
Q

A 72 year old woman was admitted 6 days ago after a fall, she has had a dynamic hip: screw inserted. She now has evidence of a hospital acquired pneumonia. Which is the most likely causative organism?
Escherichia coli.
Haemophilus influenza
Legionella pneumophila
Mycoplasma pneumoniae
Streptococcus pneumoniae

A

Escherichia coli

Escherichia coli is a common Gram-negative bacterium that can cause hospital-acquired infections, including pneumonia, particularly in patients with urinary catheters or other healthcare-associated risk factors.

326
Q
A